USMLE-Step-1-Recall-pdf.pdf

September 27, 2017 | Author: Jayanti Neogi Sardar | Category: Aorta, Heart, Atrium (Heart), Esophagus, Vagus Nerve
Share Embed Donate


Short Description

Download USMLE-Step-1-Recall-pdf.pdf...

Description

USMLE Buzzwords for the Boards THIRD EDITION

Brent A. Reinheimer

Q&. Wolters Kluwer Lippincott Williams &Wilkins

Lippincott Williams & Wilkins

A u 1 year in duration Abnormal identification with the deceased Suicidal tendencies 53

54 USMLE Step I Recall

What is the most common form of elderly abuse?

Nearly 50% of all reported cases of elderly abuse are due to neglect. Physical, psychological, and financial are other forms of elderly abuse with an overall prevalence rate of 5%-10%!

Do newborns perceive a three-dimensional world?

It is believed that newborns do perceive a three-dimensional world, because in experiments they react defensively when an object is thrust in front of their faces.

Is either ego-syntonic or ego-dystonic homosexuahty pathologic?

Homosexuality is only considered pathologic if the person is ego-dystonic.

In which syndrome does a person present with intentionally produced physical ailments with the intent to assume the "sick role"?

Miinchhausen syndrome (factitious disorder)

What term is defined as a patient unconsciously placing his or her thoughts and feelings onto the physician (in a "caregiver/ parent" role)?

Transference. When it is from the physician to the patient it is called countertransference.

What scale separates things into groups without defining the relationship between

Nominal scale (categoric) (e.g., male or female)

them? Name the area of the cerebral cortex with the function described: Speech, critical for personality, concentration, initiating and stopping tasks (do one task and begin a new task without completion of the first), abstract thought, and

Frontal lobe

Behavioral Sciences 55 memory and higher order mental functions Language, memory, and emotion (Hint: Central nervous system [CNS] herpes virus infects here commonly.)

Temporal lobe

Intellectual processing of sensory information, with the left (dominant) processing verbal information and the right processing visual-spatial orientation

Parietal lobe

Initiation and control of movements

Basal ganglia

Skill-based memory, verbal recall, balance, and refined voluntary movements

Cerebellum

Important for REM sleep and where the NE pathway begins

Pons

Motivation, memory, emotions, violent behaviors, sociosexual behaviors, and conditioned responses

Limbic system

Recall of objects, distances, and "scenes"; visual input processed here

Occipital lobe

In what organ system would you attempt to localize a sign for "shaken baby syndrome"? What do you look for?

Look for retinal hemorrhages.

56 USMLE Step I Recall

What part of the autonomic nervous system (ANS) is affected in the biofeedback model of operant conditioning? Name the anxiety defense mechanism described below: Separating oneself from the experience; the facts are accepted but the form is changed for protection

The biofeedback model is based on parasympathetic nervous system.

Dissociation

Explanations used to justify unacceptable behaviors

Rationalization

Outburst to cover up the true feelings underneath (emotion is covered, not redirected)

Acting out

Using an outlet for emotions (stuff flows downhill)

Displacement

Fact without feeling (la belle indifference)

Isolation of affect

Replacing normal affect with "brain power"

I ntellectualization

Unconsciously forgetting (forgetting that you forgot something!)

Repression

Fixing impulses by acting out the opposite of an unacceptable behavior

Undoing

Setting up to be let down (It is unconscious; if conscious, you're just rude.)

Passive-aggressive

Behavioral Sciences 57

A complete opposite expression of your inward feeling (arguing, all the time, with someone you are attracted to when your feelings have not been known) True or false—in a positively skewed curve the mean > mode

Reaction formation

False; in positively skewed distributions the mode > median > mean. (Remember to name a "skewed" distribution: The tail points in the direction of its name. Positive skew tails point to the positive end of a scale.)

3 ^

What is the formula to calculate IQ?

(MA/CA) X 100 = IQ score, where MA = mental age and CA = chronologic age

What types of foods should be avoided in patients taking MAOIs? Why?

Foods rich in tyramine should be avoided (e.g., cheese, dried fish, sauerkraut, chocolate, avocados, and red wine). Hypertensive crisis occurs when tyramine and MAOIs are mixed.

What neuropsychologic test shows nine designs to the patient, then asks him oilier to recall as many as he or she can?

Bender Visual Motor Gestalt Test

Name the immature defense mechanisms described below: Taking the beliefs, thoughts, and other external stimuli and making it a part of the self. (Hint: If it's done consciously it is called imitation.)

Introjection (a sports fan is a good example)

58 USMLE Step I Recall

Returning to an earlier stage of development (e.g., enuresis is a form)

Regression

Inability to remember a known fact (aware you are forgetting)

Blocking (I call it a brain fart!)

Psychic feelings are converted to physical symptoms

Somatization

What disorder is described as having: Unconscious symptoms with unconscious motivation?

Somatoform disorder

Conscious symptoms with conscious motivation?

Malingering

Conscious symptoms with unconscious motivation?

Factious disorder

What naturally occurring substances mimic the effects of opiates?

Enkephalins

Name the type of seizure in which the person smells "burning rubber," has hallucinations and illusions, demonstrates automatism after aura, experiences focal sensory perceptions, and loses consciousness.

Complex partial seizure

What is the most prevalent sexual disorder in men?

Secondary impotence

Behavioral Sciences 59

What somatoform disorder is described as: Having a F:M ratio of 20:1, onset before age 30, and four pains (two gastrointestinal, one sexual, and one neurologic)?

Somatization disorder

La belle indifference, suggestive of true physical ailment because of alteration of function?

Conversion disorder

Unrealistic negative opinion of personal appearance and seeing self as ugly?

Body dysmorphic disorder

Preoccupied with illness or death, persisting despite reassurance, >6-month duration?

Hypochondriasis (They will begin with, "I think I have. . . .")

Severe, prolonged pain that persists with no cause being found, which disrupts activities of daily living?

Somatoform pain disorder

Night terrors are associated with what stage of sleep?

NREM sleep. Night terrors are "dreams" that we are unable to recall.

What type of mortality rate is defined as the number of deaths: In the population?

Crude mortality rate

From a specific cause per population?

Cause-specific mortality rate

From a specific cause per all deaths?

Proportionate mortality rate

From a specific cause per number of persons with the disease?

Case fatality rate

60 USMLE Step I Recall

What is the leading cause of school dropouts among girls? What are the pharmacologic effects seen sexually with: Otj-Blockers?

Pregnancy

Impaired ejaculation

Serotonin?

Inhibited orgasm

Beta blockers?

Impotence

Trazodone?

Priapism

Dopamine agonists?

Increased erection and libido

Neuroleptics?

Erectile dysfunction

What subtype of schizophrenia is characterized by: Child-like behaviors, unorganized speech and behaviors, poor grooming, and incongruous smiling and laughing; carries the worst prognosis?

Disorganized schizophrenia

Patient being stuporous and mute, echopraxia and automatic obedience, and waxy flexibility with rigidity of posture?

Catatonic schizophrenia

Delusions of persecution and/or grandeur and auditoiy hallucinations; is late onset and carries the best prognosis?

Paranoid schizophrenia

What is the tenn to describe jumping from one topic to the next without any connection?

Loose association

Behavioral Sciences 61

In biostatistics, what are the three criteria required to increase powerr?

1. Large sample size 2. Large effect size 3. Type I error is greater

What is the teratogenic effect associated with lithium?

Epstein-cardiac anomaly of the tricuspid valve

What form of anxiety appears at 8-12 months, disappears at 20-24 months, and is seen when an infant is taken away from a caregiver?

Separation anxiety

On what part of the body does sleep deprivation have the greatest effect?

Cerebral cortex

Can parents withhold treatment from their children?

Yes, as long as the illness is not limb or life saving. If illness is critical or there is an immediate emergency, treat the child.

What is the most widely used class of antidepressants?

Selective serotonin reuptake inhibitors (SSRIs)

If the occurrence of one event precludes the occurrence of another event from happening, their probabilities are combined by what method?

Addition (They are mutually exclusive events.)

What is the first formal IQ test that is still used today for children aged 2-18?

The Stanford-Binet Scale, developed in 1905, is useful in the very bright, the impaired, or children 2 years?

Cyclothymia (nonpsychotic bipolar). Patients are ego-syntonic.

What are the five major anticholinergic side effects? (Hint: "Dry as a bone, red as a beet, blind as a bat, mad as a hatter, and hot as a hare")

Dry mouth Blurry vision Delirium Constipation Urinary retention

The DSM-IV-TR is scored on the basis of five axes of diagnosis. On what axis would you place: Psychosocial and environmental problems ("stressors")?

Axis IV

Medical or physical ailments?

Axis III

Personahty and mental disorders?

Axis II

Global assessment of function?

Axis V

Clinical disorders (e.g.. schizophrenia) ?

Axis I

Recun-ent and persistent pain before, after, or during sexual intercourse is known as?

Dyspareunia. It is a common complaint in women who have been raped or sexually abused.

Behavioral Sciences 81

What neurotransmitter is associated with sedation and weight gain?

Histamine

Is masturbation considered an abnormal sexual practice?

No. It is abnormal only if it interferes with normal sexual or occupational function.

What three circumstances allow a child to be committed to institutional care?

1. The child poses an imminent danger to self or others. 2. The child is unable to care for him- or herself daily (at the appropriate developmental level). 3. The parents/guardians have absolutely no control over the child, or will not promise to ensure the child's safety even though refusing hospitalization.

True or false—only men have refractory sexual periods.

Sad but true. Some women can have multiple successive orgasms.

Describe the normal bereavement period.

The person identifies with the deceased. The person has low suicidal tendencies. The period lasts for < 6 months.

What is defined as a general estimate of the functional capacities of a human?

IQ

What is the application of an unavoidable stimulus followed by an animal's failure to cope (when coping would odierwise be possible) known as?

Learned helplessness (e.g., Skinner box)

What ll-amino acid peptide is the neurotransmitter of sensoiy neurons that conveys pain from the periphery to the spinal cord?

Substance P (Opiates relieve pain, in part, by blocking substance P.)

In the classical conditioning model, when a behavior is

Stimulus generalization must stop. (Pairing of the unconditioned stimulus

82 USMLE Step I Recall

learned, what needs to occur in order to break the probability that a response will happen? Name the stage of sleep with the EEG pattern described below: Disappearance of alpha waves, appearance of theta waves

and die conditioned stimulus needs to cease.)

Stage 1

Delta waves

Stages 3 and 4

Sawtooth waves, random low-voltage pattern

REM

Alpha waves

Being awake

Sleep spindles, K-complexes

Stage 2

Most sleep time is spent in what stage of sleep?

What Freudian psyche component is described as: The urges, sex aggression, and "primitive" processes?

Stage 2, which accounts for approximately 45% of total sleep time, with REM occupying 20%

Id (pleasure principle)

The conscience, morals, and beliefs (middle of the road)?

Superego

Reality, rational, and language based?

Ego

What is the term to describe the average?

Mean

What disorder is defined by the following criteria: occurs

Enuresis (bed wetting)

Behavioral Sciences 83

during stages 3 and 4 of sleep; can't be diagnosed until 5 years of age; occurs more often in boys; occurs at least two times a week for at least 3 consecutive months; and causes dish-ess or impairment in social functioning? What statistical test compares the means of groups generated by two nominal variables by using an interval variable?

Two-way ANOVA. It allows the test to cheek several variables at the same time.

What happens to Cortisol levels in sleep-deprived individuals?

Cortisol levels increase. Lymphocyte levels decrease in sleep-deprived individuals (due to physiologic stress).

Can committed mentally ill patients refuse medical treatment?

Yes. The only civil liberty they lose is the "freedom" to come and go as tiiey please.

The proportion of truly nondiseased persons in the screened population who are identified as nondiseased refers to?

Specificity (It deals with the healthy.)

Name the area of the cerebral cortex affected by the description of the effects/symptoms/results of the lesion. Apathy, aggression, inability to learn new material, and memory problems Apathy, poor grooming, poor ability to think abstractly, decreased drive, and poor attention span (Hint: If in the

Limbic system

Dorsal prefrontal cortex

84 USMLE Step I Recall

dominant hemisphere, patient will develop Broca's aphasia.) Euphoria, delusions, thought disorders, Wernicke's aphasia, and auditory hallucinations (Hint: It is in the left hemisphere.)

Dominant temporal lobe

Agraphia, acalculia, finger agnosia, and right-left disorientation

Dominant parietal lobe (Gerstmann's syndrome)

Withdrawn, fearful, explosive moods, violent outbursts, and loss of inhibitions

Orbitomedial frontal lobe

Denies illness, hemineglect, and construction apraxia (e.g., can't arrange match sticks)

Nondominant parietal lobe

Denies being blind, cortical blindness

Occipital lobe (Anton's syndrome if it is due to bilateral posterior cerebral arteiy occlusions)

Dysphoria, irritability, and musical and visual abilities decreased

Nondominant temporal lobe

If the family member of a patient asks you to withhold information, would you?

For the USMLE Step I the answer is no, but in reality, if the information you are going to tell your patient would do more harm than good, withhold. This is very rare but it does occur.

What scale is graded into equal increments allowing one to not only see if there is a difference, but also by how much?

Interval scale (a ruler, for example)

Behavioral Sciences 85

Increased levels of what neurotransmitter, in the hippocampus, decrease the likelihood of learned helplessness?

Increased Y-aminobutyric acid (GABA) levels decrease the likelihood of learned helplessness.

Which drug is used to treat the respiratory depression associated with an overdose of opioids?

Naloxone or naltrexone

The probabUity that a person with a negative test result is truly disease free refers to what value?

Negative predictive value

What percentage of suicides occur over the age of 65?

25% (The elderly attempt suicide less often, but succeed more often.)

What is the term for the alteration of the subjects' behavior because they are being studied?

Hawthorne effect

What medication is used in alcoholics to help prevent relapse by decreasing the glutamate receptors' activity?

Acamprosate (The number of glutamate receptors increases with chronic alcohol abuse.)

What happens to norepinephrine (NE) levels in: Major depression?

Bipolar disorder?

What fonn of anxiety is seen in the presence of unfamiliar people, appears at 6 months, peaks at 8 months, and disappears by 1 year

of age?

They decrease (5-HT and dopamine levels do the same). They increase (5-HT and dopamine levels do the same). Stranger anxiety

86 USMLE Step I Recall

What five things are checked in the APGAR test?

1. Skin color 2. Heart rate 3. Reflexes 4. Muscle tone 5. Respiratory rate APGAR = Appearance, Pulse, Grimace, Activity, Respiration

What is the most common cause of death in black males aged 15-24?

Homicide. It is also the leading cause of death in black females aged 15-24.

The probability that a person with a positive test result is truly positive refers to what value?

Positive predictive value

What type of randomized clinical trial (RCT) is least subjective to bias?

Double-blind study. It is the most scientifically rigorous study known.

What treatment used to treat anxiety and phobias is slow, stepwise, and based on counterconditioning?

Systemic desensitization

What types of symptoms in schizophrenia are associated with: Dopamine receptors?

Muscarinic receptors

(ACh)?

Type I symptoms (positive); schizophrenics have them; otherwise healthy persons do not. Type II symptoms (negative); otherwise healthy persons have them; schizophrenics do not.

What happens to incidence and prevalence if: New effective treatment is initiated?

No change in incidence; prevalence decreases

New effective vaccine gains widespread use?

Decrease in incidence; prevalence decreases

Behavioral Sciences 87

Number of persons dying from the condition increases?

No change in incidence; prevalence decreases

Additional federal research dollars are targeted to a specific condition?

No change in incidence; no change in prevalence

Behavioral risk factors are reduced in the population at large?

Decrease in incidence; decrease in prevalence

Long-term survival rates for the disease are increased?

No change in incidence; increase in prevalence

What major side effect of neuroleptics is characterized by pill rolling, shuffling gait, and tremors that abate during sleep?

Tardive dyskinesia. It persists even after treatment is discontinued and has no treatment. Focus is on monitoring for side effects and prevention.

What type of reinforcement strengthens each response and involves fast learning and fast extinction?

Continuous reinforcement

What classical conditioning therapy or modification is described as: Pairing noxious stimuli to an inappropriate behavior?

Aversive conditioning

Forcing patients to confront their fears by being exposed to them until they are extinguished?

Exposure

A hierarchy of "fears" (from least to most) is triaged, and then muscle

Systematic desensitization

88 USMLE Step I Recall

relaxation techniques are taught in the presence of that fear until patients are not afraid anymore? What chromosome is autism linked to? Name the cluster A personality disorder described below: Is odd and strange, has magical flunking, is socially isolated and paranoid, lacks close friends, and has incongruous affect

Chromosome 15

Schizotypal

Is socially withdrawn, seen as eccentric, but happy to be alone

Schizoid

Has baseline mistrust, carries grudges, is afraid to "open up," uses projection as defense mechanism, and lacks hallucinations or delusions

Paranoid

Having a deficiency or an absence of sexual fantasies or desires is referred to as?

Hvpoactive sexual desire disorder

Which drug is used to treat opiate withdrawal, attention deficit/ hyperactivity disorder (ADHD), and sometimes Tourette's syndrome?

Clonidine

Is spousal abuse a mandatory reportable offense?

No, it is not a mandatoiy reportable offense (if you can believe it). Child and elderly abuse are mandatory reportable offenses.

Behavioral Sciences 89

What pineal honnone's release is inhibited by daylight and increased dramatically during sleep?

Melatonin. It is a light-sensitive hormone that is associated with sleepiness.

The following characteristics describe what form of dementia: onset between the ages of 40 and 50, rapidly progressive, caused by a prion, and fatal within 2 years?

Creutzfeldt-Jacobs disease. Patients initially present with vague somatic complaints and anxiety rapidly followed by dysarthria, myoclonus, ataxia, and choreoathetosis.

What statistical test, using nominal data only, checks if two variables are independent events?

Chi-square (When in doubt and you have nominal data, use chi-square.)

What are the five pieces of information needed to be considered "fully informed" for consent?

1. 2. 3. 4.

When does most of the NREM sleep (stages 3 and 4) occur, in the first or second half of sleep?

The deepest sleep levels (stages 3 and 4) occur more often in the first half of sleep.

What is the name of the most widely used personality test consisting of 550 true/ false questions, in which the overall score is more important than an evaluation on an individual scale?

Minnesota Multiphasic Personality Inventoiy (MMPI)

A 95% confidence score means the z score is equal to what?

Benefits of the procedure Purpose of the procedure Risks of the procedure The "nature" of the procedure (what you are doing) 5. The alternative to the procedure and its availability (Don't forget the last one; this is where physicians get in trouble.)

90 USMLE Step I Recall

If y o u make a mistake a n d t h e patient doesn't find out anyway, should y o u disclose this to your patient?

Yes, you must take responsibility for the bad as well as the good.

At what a g e d o e s a child develop: At birth (reflex) E n d o g e n o u s smile? 8 weeks (response to a face) E x o g e n o u s smile? 12-16 weeks (in response to mom's face) Preferential smile? T h e difference b e t w e e n t h e highest and the l o w e s t score in a population is known as?

Range

Can a patient refuse lifesaving treatment b e c a u s e o f religious reasons?

Yes. (Remember: a Jehovah's witness who refuses blood transfusions.)

What form of d e p r e s s i o n is d u e to abnormal m e t a b o l i s m o f melatonin?

Seasonal affective disorder (treat with bright light therapy)

What are the four m o s t common defense mechanisms used by obsessive-compulsive persons?

1. 2. 3. 4.

At what age d o children b e g i n to u n d e r s t a n d t h e irreversibility of d e a t h ?

At 8-9 years of age. Prior to this age, they view death as a form of punishment.

W h e n d o e s most of t h e R E M s l e e p occur, in t h e first or s e c o n d half of s l e e p ?

REM sleep occurs more often in the second half of sleep. The amount of REM sleep progressively increases as the night goes on.

What h e m i s p h e r e is u s e d for l a n g u a g e and (for most p e o p l e ) is the d o m i n a n t hemisphere?

Left hemisphere (Left = Language)

Isolation of affect Undoing Reaction formation Intellectualization

Behavioral Sciences 91

True or false—being college educated increases a man's risk of having premature ejaculation.

Tme; higher level of education, stressful marriages, early sexual experiences in the back of a car, or sex with a prostitute all increase the risk of premature ejaculation.

What is the "key issue" surrounding teenagers' maturation?

Formation of an identity through issues of independence and rebellion; they define who they are.

What Alzheimer's disease dementia has a defect in chromosome 4, onset between the ages of 30 and 40, choreoathetosis, and progressive deterioration to an infantile state?

Huntington's chorea (death in 15-20 years often via suicide)

What type of questions should you begin with when a patient seeks your medical opinion?

It is best to begin with open-ended questions, allowing patients to describe in their own words what troubles them. You can then move to closed-ended questions when narrowing the diagnosis.

True or false—marriage makes a child (13 years old "caring" for him- or herself also serve to emancipate a child.

What is the number one predisposing factor for vascular dementia?

Hypertension

Kaiser-Fleischer rings, abnormal copper metabolism, and ceruloplasmin deficiency characterize what disease, which may include symptoms of dementia when severe?

Wilson's disease (Remember: chromosome 13 and hepatolenticular degeneration)

What hypothesis states that the findings of a test are as a result of chance?

Null hypothesis (what you hope to disprove)

92 USMLE Step I Recall

Name the aphasia based on the characteristics described below: Nonfluent speech, telegraphic and ungrammatical, lesion in Brodmann's area 44, comprehension is unimpaired

Broca's aphasia

In the prefrontal cortex, can't speak spontaneously, ability to repeat is unimpaired

Transcortical aphasia

Lesion is in the parietal lobe or arcuate fibers because the connection between Broca's and Wernicke's area is severed, word comprehension is preserved but can't write or speak the statement (can't tell you what you said)

Conduction aphasia

Both Broca's and Wernicke's areas are damaged due to lesion in the presylvian speech area, trouble repeating statements, poor comprehension with telegraphic speech

Global aphasia

Lesion in Brodmann area 22, comprehension impaired, incoherent fluent speech (rapid), verbal paraphrasias, trouble repeating statements

Wernicke's aphasia

Behavioral Sciences 93

What is the foremost cause of cancer death in both men and women?

Lung cancer

What honnone's release is strongly associated with stage 4 sleep?

Growth hormone (GH). The largest output of GH in a 24-hour period is during stage 4 sleep,

What are the four exceptions to "informed" consent?

1. Incompetent patient (determined by the courts) 2. Therapeutic privilege (in the best interest of the patient when he or she is unable to answer) 3. Waiver signed by the patient 4. Emergency situations

What syndrome is characterized by sweating, insomnia, nausea, diarrhea, cramps, delirium, and generalized restlessness secondaiy to monoamine oxidase inhibitor (MAOI) and SSRI combination?

Serotonin syndrome. It is also associated with high doses and MAOI and synthetic narcotic combinations (e.g., Ecstasy). Treatment consists of decreasing SSRI dosage, removing the causative agent, and giving cyproheptadine.

What is the name of the hypothesis you are trying to prove?

Alternative hypothesis (what is left after the null has been defined)

What happens to dopamine levels when we awaken?

Dopamine levels rise with waking and are associated with wakefulness.

What stage of sleep are bruxisms associated with?

Teeth grinding is associated with stage

What is the name of the benzodiazepine antagonist used in the treatment of an overdose?

Fli:

What is the most commonly reported sexually transmitted disease (STD)?

Gonorrhea

2 sleep. ¡nil

94 USMLE Step I Recall

What do you do with the null hypothesis if the P value is >0.05?

Do not reject the null hypothesis.

What are the three stages that children (aged 7 montiis to 5 years) go through when they are separated from a primary caregiver for a prolonged period of time?

1. Protest 2. Despair 3. Detachment

The proportion of truly diseased persons in the screened population who are identified as diseased refers to?

Sensitivity (It deals with the sick.)

What is the most common primary diagnosis resulting in an office visit for males? For females?

For males, hypertension; for females, pregnancy

Chapter 3

Biochemistry

What is the drag of choice in treating a patient with hyperuricemia due to overproduction of uric acid?

Allopurinol

In what position of the codon and the anticodon does the wobble hypothesis take place?

3'end of the codon (third position) on mRNA and 5' end of the anticodon (first position) on tRNA

True or false—DNA polymerases can correct mistakes, whereas RNA polymerases lack this abdity.

Tme; DNA polymerases have 3'-5' exonuciease activity for proofreading.

Biochemistry 95

How many ATPs/glucose are generated from glycolysis in red blood cells (RBCs)?

Two ATPs because RBCs only use anaerobic metabolism

What is the function of glycogen stores in the muscle?

Muscle glycogen stores in the muscle are utilized as an energy reserve for the muscle, whereas liver glycogen stores are utilized in hypoglycemia.

How many base pairs upstream is the prokaryotic TATA box promoter?

There are two bacterial promoter regions upstream, the TATA box is - 1 0 base pairs upstream and the -35 promoter site is self-explanatory.

What three AAs must be avoided in the diet in patients with maple syrup urine disease?

Isoleucine, leucine, and valine

What cofactor is needed for lysyl oxidase?

Cu2'

What pattern of genetic transmission is characterized by the following: there is no transmission from M, it is inherited maternally, and both sons and daughters of affected F can be affected by the disease?

Mitochondrial inheritance

What protein binds to the promoter region in eukaiyotes to initiate transcription?

Transcription factor (TF) II D

What pattern of genetic transmission is characterized by the following: both M and F are affected, there is no M-to-M transmission, affected M passes trait to all daughters, it occurs eveiy generation, affected F

X-linked dominant

96 USMLE Step I Recall

passes trait to both sons and daughters, and only one mutant allele is needed for disease to be present? What are the three "stop" codons?

1. UAA 2. UAG 3. UGA

What enzyme catalyzes the rate-limiting step in glycogenolysis?

Glycogen phosphoiylase

What amino acid undergoes N-glycosylation?

Asparagine

What X-linked recessive disorder is characterized by hyperuricemia, spastic cerebral palsy, mental retardation, and self-mutilation?

Lesch-Nyhan syndrome

Aldose reductase converts galactose to what?

Galactitol

Why are erythrocytes unable to use fatty acids as a source of energy?

They lack mitochondria (Remember fatty acid oxidation occurs in the mitochondria.)

What is the primary endproduct of purine synthesis?

Inosine monophosphate (IMP)

Is the coding or the template strand of DNA identical to mRNA (excluding the T/U difference)?

The coding strand is identical to mRNA and the template strand is complementary and antiparallel.

What enzyme is used to seal the nicks after the complementary "sticky end" bases pair with each other?

DNA ligase

What enzyme is deficient in alcaptonuria?

Homogentisic acid

Biochemistry 97

What elements make up a nucleoside?

A base and a sugar

What AA is the major carrier of nitrogen byproducts from most tissues in the body?

Glutamine

What amino acid is attached to the 3' end of the tRNA in eukaryotes?

Methii

What enzyme catalyzes the rate-limiting step of the urea cycle?

Carbamoyl phosphate synthetase I

What protein prevents the ssDNA from reannealing during DNA replication?

Single-stranded DNA-binding protein

What enzyme deficiency will result in an increase in blood ammonia, but no increase in uracil concentrations?

Carbamoyl-phosphate synthetase

What enzyme is deficient in a patient 2 years of age or younger who presents with vomiting, lethargy, coma, hypoketosis, and hypoglycemia following a fast of more than 12 hours?

Medium-chain acyl dehydrogenase

In what organelle does the tricarboxylic acid (TCA) cycle occur?

Mitochondria

What are the three posttranscriptional modifications ?

1. 7-methyl guanine cap on the 5' end 2. Addition of die poly(A) tail to the 3' end 3. Removal of introns

During a prolonged fast, there is a shift from glucose to ketones as an

It spares protein degradation because protein has no storage fonn.

98 USMLE Step I Recall

energy source. How does this affect protein degradation? What three substances stimulate glycogenolysis?

1. Ca2+:calmodulin ratio 2. Epinephrine 3. Glucagon

What is the site of action of cycloheximide?

Peptidyl transferase (60S)

In what disease are lysosomal enzymes released into the extracellular space where an accumulation of inclusion bodies compromises the cell's function?

I-cell disease

Would a G-C- or an A-Trich dsDNA sequence have a higher melting point? Why?

G—C-rich sequence because it has three hydrogen bonds (whereas A-T has two hydrogen bonds), resulting in higher melting points.

What is the only organ in the body that can produce ketone bodies?

The liver (in the mitochondria)

What three amino acids are used to synthesize the purine ring?

1. Glycine 2. Aspartate 3. Glutamine

True or false—the urea cycle takes place in both the cytoplasm and the mitochondria.

Tme; remember, carbamoyl phosphate Synthetase mid ornithine transcarbamoylase are mitochondrial enzymes.

What honnone is activated in adipose tissue when blood glucose levels decrease?

Hormone-sensitive lipase

What enzyme is absent in the liver so that ketogenolysis cannot occur?

Thiophorase

Biochemistry 99

What two amino acids have a pKa of 10?

1. Lysine 2. Tyrosine

What causes transcription to stop in eukaiyotes?

The poly(A) site on the DNA

Name the type of mutation described below: The deletion or addition of a base

Frameshift

New codon specifies a different AA

Missense

Unequal crossover in meiosis with loss of protein function

Large segment deletions

New codon specifies for the same AA

Silent

New codon specifies for a stop codon

Nonsense

How many ATPs are generated per each acetyl CoA?

12 ATPs/acetyl CoA, that enter the TCA cycle

Methylating uracil produces what pyrimidine base?

Thymine

What are the two precursors of heme?

1. Glycine 2. Succinyl-CoA

True or false—methylation of bacterial DNA prevents restriction endonuclease from cutting its own chromosomes.

Tme; restriction endonucleases only cut unmethylated DNA.

True or false—histidine activates the histidine Operon.

False; the histidine operon is activated when there are low intracellular levels of histidine.

100 USMLE Step I Recall

The number of trinucleotide repeats increasing with successive generations and correlating with increased severity of the disease is known as?

Anticipation, associated with fragile X syndrome and Huntington's disease, is also associated with a decrease in onset of age.

Is linolenic acid an omega-3 or -6 fatty acid?

Omega-3; linoleic is omega-6.

What are the five factors that constitute the pyruvate dehydrogenase complex?

1. 2. 3. 4. 5.

What enzyme deficiency results in darkening of the urine when exposed to air?

Homogentisate oxidase deficiency is seen in patients with alcaptonuria.

What eukaryotic translation enzyme is associated with the following: Initiation?

Thiamine pyrophosphate (TPP) Lipoic acid CoASH Flavin adenine dinucleotide (FAD) NAD

eIF-2 in the P site

Elongation?

eEF-1

Termination?

No enzymes are needed; when the stop codon reaches the A site, it results in termination.

What pattern of genetic transmission is characterized by the following: both M and F are affected, M-to-M transmission may be present, both parents of an affected child must be carriers, it skips generations, two mutant alleles are needed for disease to be present, and there are affected children from unaffected adults?

Autosomal recessive

Biochemistry 101

What is the most common methylated base?

Cytosine

What part of the 30S ribosome binds to the Shine-Dalgarno sequence?

16S subunit

What are the three tissues where triacylglycerols are produced?

1. Liver 2. Muscle 3. Adipose tissue

What enzyme is deficient in hereditary protoporphyria?

Ferrochelatase

What three substrates control the enzyme PEPCK for the conversion of OAA to pyruvate in the cytoplasm?

1. Cortisol (stimulates PEPCK) 2. Glucagon 3. GTP

What bonds are broken by endonucleases?

Internal 3', 5' PDE bonds

What is the rate-limiting step of: Glycolysis? Gluconeogenesis?

PFK-1 Pyruvate carboxylase

TPP is associated with what three enzymes?

1. OC-Ketoglutarate dehydrogenase 2. Pyruvate dehydrogenase 3. Transketolase

All the carbons in a fatty acid are derived from what source?

Cytoplasmic acetyl CoA that left the mitochondria as citrate

Why is the liver unable to metabolize ketone bodies?

Hepatocytes lack the enzyme succinyl CoA acetoacetyl CoA transferase (thiophorase).

What enzyme catalyzes the rate-limiting step in pyrimidine synthesis?

Aspartate transcarbainylase

102 USMLE Step I Recall

What pyrimidine base is found: Only in RNA?

Uracil

Only in DNA?

Thymine

In both DNA and RNA?

Cytosine

If a sample of DNA has 30% T, what is the percentage of C?

Solved as: 30% T + 30% A = 60%; therefore, C + G = 40%, then C = 20% and G = 20% (Example of Chargaffs mle)

What substrate is built up in Niemann-Pick disease?

Sphingomyelin

What AA is broken down into N 2 0 , causing an increase in cyclic guanosine monophosphate (cGMP) of smooth muscle, resulting in vasodilation?

Arginine

What amino acid has a pKa of 13?

Arginine

What is the mode of inheritance in which a trait is seen in every generation and is passed on only by females?

Mitochondrial inheritance

The process of converting a dsDNA molecule to the base sequence of an ssRNA molecule is know as?

Transcription ("C" comes before "L" in the alphabet and transcription comes before transLation)

What are the eight liverspecific enzymes?

1. 2. 3. 4. 5. 6. 7. 8.

Fructokinase Glucokinase Glycerol kinase Phosphoenolpyruvate carboxykin (PEPCK) Pyruvate carboxylase Galactokinase Fmctose-l,6-biphosphate Glucose-6-phosphate

Biochemistry 103

What amino acid is a phenol?

Tyrosine

What are the two inhibitors of complex I of the ETC?

1. Rotenone 2. Amytal (barbiturates)

What carries triacylglycerols and cholesterol from the diet?

Chylomicrons

What is the only way to increase the V of a

Increase the concentration of enzymes

ma.\

reaction? What enzyme is deficient in selective T-cell immunodeficiency?

Purine nucleoside phosphoiylase

What is the drug of choice in treating a patient with hyperuricemia due to underexcretion of uric acid?

Probenecid (It is a uricosuric agent.)

What substrate concentration is required to produce 1/2 V ? What enzyme is deficient in maple syrap urine disease?

Branched-chain keto acid dehydrogenase

What type of jaundice is seen in Rotor's syndrome?

Conjugated (direct) hyperbilirubinemia

What hormone stimulates glycogen synthesis?

Insulin

What is the rate-limiting enzyme on glycolysis?

Phosphofructokinase-1 (PFK-1) and costs one ATP

What needs to be supplemented in patients with medium-chain acyl CoA dehydrogenase deficiency (MCAD)?

Short-chain fatty acids

104 USMLE Step I Recall

How many ATPs/glucose are produced by pyruvate dehydrogenase?

Six ATPs (Remember, two pyruvat are produced per glucose and two NADHs result when producing acetyl CoA, so therefore, there are six ATPs.)

A deficiency in what enzyme causes a decrease in oxidoreductase activity in neutrophils?

G-6-PD

What enzyme of the TCA cycle also acts as complex II of the electron transport chain (ETC)?

Succinate dehydrogenase

What enzyme of pyrimidine synthesis is inhibited by: 5-FU?

Thvmidylate synthase

Methotrexate?

Dihydrololate reductase

Hydroxyurea?

Ribonucleotide reductase

What disease has a genetically low level of uridine diphosphate (UDP) glucuronate transferase, resulting in elevated free unconjugated bilirubin?

Gilbert's syndrome

Where does translation occur?

In the cytoplasm (mRNA to protein)

Deaminating cytosine produces what pyrimidine base?

Uracil

What are the two ways that nitrogen can enter into the urea cycle?

1. Aspartate 2. Carbamoyl PO,

What two amino acids have apKaof4?

1. Aspartic acid 2. Glutamic acid

Biochemistry 105

What DNA polymerase removes RNA primers and fills in the "gaps" in a 5'-3' direction?

DNA polymerase I

What two posttranscriptional enzymes in collagen synthesis require ascorbic acid to function properly?

Prolyl and lysyl hydroxylases

Name three purine bases that are NOT found in nucleic acids?

Xanthine, hypoxanthine, theophylline, theobromine, caffeine, and uric acid are all purines.

What two AAs do not have more than one codon?

Methionine (start) and tryptophan are the only two AAs that have one codon.

What enzyme is blocked by disulfiram?

Aldehyde dehydrogenase

Name the phase of the eukaiyotic cell cycle described as the: Period of cellular growth (translation and transcription) before DNA synthesis

Gl phase (gap 1)

Period of cellular growth (translation and transcription) after DNA synthesis

G2 phase (gap 2)

Period of DNA replication (preparing for mitosis)

S phase

Cells cease replicating (i.e., nerve cell)

GO phase

What toxin ADP-ribosylates via G. to increase cAMP?

Pertussis toxin

i

The addition of D-2, 3-bisphosphoglycerate

Shifts it to the right

106 USMLE Step I Recall

(D-2, 3-BPG) to HbA does what to the 0 2 saturation

What are the three diseases of sphingolipids?

1. Niemann-Pick disease 2. Gauchers disease 3. Tay-Sachs disease

What DNA replication enzyme seals the two Okazaki fragments together?

DNA ligase

What apoprotein is required for the release of chylomicrons from the epithelial cells into the lymphatics?

apo B-48

What bonds are broken by ertonucieflses?

External 3', 5' phosphodiester (PDE) bonds

What is the start codon and what does it code for in eukaryotes? Prokaryotes?

The one start codon, AUG, in eukariptes codes for methionine and in prokaryotes for formylmethionine.

What apoprotein on HDL activates lecithin-cholesterol acyltransferase (LCAT)?

apo A-l

How many covalent bonds per purine-pyrimidine base pairing are broken during denaturation of

Zero; denaturation of dsDNA breaks hydrogen bonds, not covalent bonds.

dsDNA? What form of DNA is described as: The most common form found in nature and a right-handed double helix? A left-handed double helix that occurs in G—C-rich sequences?

B-DNA (also known as Watson-Crick DNA)

Z-DNA

Biochemistry 107

What enzyme of purine synthesis is inhibited by allopurinol and 6-mercaptopurine?

PRPP aminotransferase

What protein carries free fatty acids to the liver?

Albumin

How many bases upstream is the eukaryotic TATA box promoter?

There are two eukaryotic upstream promoters; the TATA box is located - 2 5 base pairs upstream and the CAAT box is located -75 bases upstream.

What enzyme catalyzes the rate-limiting step in fatty acid synthesis?

Acetyl CoA carboxylase

What two tissues prefer ketone bodies over glucose?

1. Heart muscle 2. Renal cortex

What is the primary end-product of pyrimidine synthesis?

Uridine monophosphate (UMP)

On the Lineweaver-Burke plot, what type of binding has both plots crossing the x-axis in the same spot?

Noncompetitive, reversible binding (decrease in Vmux . ; Kin is the same)

What two shuttles are needed to keep NAD + in the reduced state?

Malate/aspartate and glycerol3-phosphate shuttles

The carbons for fatty acid synthesis leave the mitochondria as what compound?

Citrate, via the citrate shuttle

What determines the rate of reaction?

The energ)' of activation

Name the RNA subtype based on the following: The most abundant form of RNA in the cell

7

l-RNA

108 USMLE Step I Recall

Found only in the nucleus of eukaryotes and functions to remove introns from mRNA

snRNA

Only type of RNA that is translated

mRNA

Carries amino acids (AAs) to the ribosome for protein synthesis

tRNA

RNA molecules with enzymatic activity

Ribozymes

Found only in the nucleus of eukaiyotic cells and are precursors of mRNA

hnRNA

What enzyme is deficient in the following glycogen storage diseases: GIucose-6-phosphatase von Gierke's disease? Lysosomal oc-l,4-glucosidase Pompe's disease? Muscle glycogen Phosphorylase McArdle's disease? Hepatic glycogen Phosphorylase Hers' disease? What is needed to initiate translation?

Intrinsic factor (IF) and GTP (elF for eukaryotes)

How many ATPs are produced from cytoplasmic NADH oxidation using the malate shuttle?

Three ATPs by oxidative phosphorylation

Muscles send nitrogen to the liver as what AAs?

Alanine and glutamine

What are the two ketogenic AAs?

Leucine and lysine

Biochemistry 109

What DNA excision and repair enzyme is deficient in patients with xeroderma pigmentosum?

Excision endonuclease (which removes thiamine dimmers from DNA)

How many codons code for AAs? How many for termination of translation?

61 codons code for AAs and three codons (UAA, UGA, UAG) code for the termination of translation.

What four substances increase the rate of gluconeogenesis?

1. 2. 3. 4.

What substrate gets built up in Gaucher's disease?

Glucosyl cerebroside

What two glycolytic enzymes catalyze the substrate-level phosphorylations?

3-Phosphoglycerate kinase and pvruvate kinase; this produces two ATPs/ enzyme (total four ATPs).

How many acetyl CoAs/ glucose enter into the TCA

Two acetyl CoAs/glucose, producing 12 ATPs/acetyl CoA, resulting in a total of 24 ATPs produced from glucose (via acetyl CoA) that enter the TCA cycle

cycle?

Glucagon NADH Acetyl CoA ATP'

What type of organisms have inonocistronic mRNA?

Eukaryotes

What is the main inhibitor of pyruvate dehydrogenase?

Acetyl CoA (pyruvate to acetyl CoA)

What two enzymes of heme synthesis are inhibited by lead?

ALA dehydrogenase and ferrochelatase

What five pathways use S-adenosylmethionine (SAM) as the methyl donor?

1. 2. 3. 4. 5.

Epinephrine synthesis Phosphatidyl choline Creatine Methylation of cvtosine N-methyl cap of mRNA

MO USMLE Step I Recall

What in the human genome differs in each individual that can serve as an identification marker?

RFLP—restriction fragment length polymorphism

What is the complex needed for propionyl CoA carboxylase?

Biotin, ATP, and CO,

What is the primer for the synthesis of the second strand when producing cDNA from mRNA?

The haii-pin loop made by reverse transcriptase at the 3' end of the first strand is the primer.

What prokaryotic positioning enzyme in translation is blocked by: Tetracycline?

eEF-Tu and eEF-Ts of the 30S ribosomal subunit

Erythromycin?

eEF-G of the 50S subunit

Streptomycin?

eIF-2ofthe30S subunit

What form of bilirubin is carried on albumin?

Unconjugated (indirect)

What tumor suppressor gene prevents a cell with damaged DNA from entering the S phase?

p53 gene

What test is used to determine whether a gene is expressed?

Northern blotting

What enzyme is deficient in patients with phenylketonuria (PKU)?

Phenylalanine hydroxylase

What protein is required for the uptake of LDLs in the peripheral tissue?

Apoprotein B-100

Biochemistry I I I

What fonn of AA is found only in collagen?

Hydroxyproline

What topoisomerase makes ssDNA cuts, requires no ATP, relaxes supercoils, and acts as the swivel in front of the replication fork?

Topoisomerase I (relaxase)

What two enzymes are blocked by lead?

1. ALA dehydratase 2. Ferrochelatase

What cytoplasmic organelle carries the enzymes for elongation and desaturation of fatty acyl CoA?

Smooth endoplasmic reticulum (SER)

If the pH is more acidic than the pi, does the protein cany a net positive or negative charge?

When the pH is more acidic than the pi, it has a net positive charge, and when the pH is more basic than the pi, it has a net negative charge.

How many ATPs are produced from cytoplasmic NADH oxidation using the glycerol phosphate shuttle?

Two ATPs by oxidative phosphorylation

What are the ketogenic amino acids?

Leucine and lvsine

Regarding the Lac operon, what do the following genes code for: Z gene?

ß-Galactosidase

Y gene?

Galactoside permease

I gene?

Lac repressor protein

A gene?

Thiogalactoside transacetylase

What AA is classified as basic even though its pK is 6.5-7.0?

Histidine, due to the imidazole ring found in the "R" group, is basic.

I 12 USMLE Step I Recall

What is the size of the prokaryotic ribosome?

70S ribosomes in prokaryotes and 80S ribosomes in eukaiyotes

What hepatic enzyme phosphorylates glucose?

Glucokinase phosphorylates glucose in the liver and pancreatic ß cells, whereas hexokinase phosphorylates glucose in all the other tissues. This step costs one ATP.

What two requirements need to be met for the Lac operon to be activated?

Lactose must be present and glucose must be absent.

During a prolonged fast, why is the brain unable to utilize fatty acids?

Fatty acids cannot cross the blood-brain barrier; therefore, they cannot be utilized as an energy source.

What component of the ETC is inhibited by the following: Barbiturates?

Complex I

Antimycin A?

Cytochrome b/cl

Cyanide?

Cytochrome a/a3

Oligomycin?

Fo/Fl complex

Atractyloside?

ATP/ADP translocase

Carbon monoxide?

Cytochrome a/a3

Rotenone?

Complex I

Azide?

Cytochrome a/a.3

What fonn of continuous DNA, used in cloning, has no introns or regulatory elements?

cDNA, when it is made from mRNA

At which end of DNA are new bases added?

3' end

Biochemistry I 13

Are the following conditions associated with a negative or positive nitrogen balance: AA deficiency?

Negative

Growth?

Positive

Pregnancy

Positive

Uncontrolled diabetes mellitus (DM)?

Negative

Starvation?

Negative

Infection?

Negative

Recovery from injury?

Positive

Kwashiorkor?

Negative

What protein is required by prokaryotic RNA polymerases to initiate transcription at the promoter region of DNA?

Sigma factor

What enzyme produces an RNA primer in the 5'-3' direction, and is essential to DNA replication because DNA polymerases are unable to synthesis DNA without an RNA primer?

Primase

What attaches to protons and allows them to enter into the mitochondria without going through the ATP-generating system?

2,4-Dinitrophenol

What type of jaundice is seen in Dubin-Johnson syndrome?

Conjugated (direct) hyperbilirubinlemia (It is a transport defect.)

114 USMLE Step I Recall

How many hydrogen bonds link adenine (A) and thymine (T)? Cytosine (C) and guanidine (G)?

A and T are linked by two hydrogen bonds, C and G by three hydrogen bonds.

What happens to affinity if you increase Km?

Affinity decreases (they are inversely proportional).

What enzyme makes rRNA (barring the 5s subunit)?

RNA polymerase I

What is the rate-limiting enzyme of glycogen synthesis?

Glycogen synthase

In the mitochondria, what complex is needed in order for pyruvate carboxylase to catalyze the reaction from pyruvate to oxaloacetate (OAA)?

Biotin, ATP, and CO,

In prokaryotes, what is the term for a set of structural genes that code for a select group of proteins and the regulatory elements required for the expression of such genes?

Operon

How many ATPs are generated from glycolysis?

Eight

What lysosomal enzyme is deficient in: Gaucher's disease?

Glucocerebrosidase

Niemann-Pick disease?

Sphingomyelinase

Tay-Sachs disease?

Hexosaminidase A

What three steps of the TCA cycle generate NADH?

1. Malate dehydrogenase 2. Isocitrate dehydrogenase 3. a-Ketoglutarate dehydrogenase

Biochemistry I 15

Name the lipoprotein based on the following characteristics, apo E

Intermediate-density lipoprotein (IDL)

apo B-100

Low-density lipoprotein (LDL)

apo E, apo B-100, apo C-II

very-low-density lipoprotein (VLDL)

apo A-1, apo E, apo C-II

High-density lipoprotein (HDL)

apo E, apo C-II, apoB-48

Chylomicrons

What pattern of genetic transmission is characterized by the following: it affects only M, there is no M-to-M transmission, and the mother is usually an unaffected carrier?

X-linked recessive

What DNA replication enzyme breaks the hydrogen bond of base pairing, fonning two replication forks?

Helicase (requires adenosine triphosphate [ATP] for energy)

In prokaryotes, what is the name of the RNA sequence that ribosomes bind to so translation can occur?

Shine-Dalgarno sequence

Which organisms have polycistronic mRNA?

Prokaryotes Polycistronic and prokaryotes bodi start with P.

What complex of the ETC contains Cu 3+ ?

Complex 4

What enzyme of the TCA cycle catalyzes the substratelevel phosphorylation?

Succinyl CoA synthetase

116 USMLE Step I Recall

In what disease is there a genetic absence of UDP glucuronate transferase, resulting in an increase in free unconjugated bilirubin?

Ciigler-Najjar syndrome

What two amino acids disrupt an a-helix?

1. Glycine 2. Proline

What enzyme catalyzes the rate-limiting step in the TCA cycle?

Isocitrate dehydrogenase

What are the five AAs that are both ketogenic and glucogenic?

Isoleucine, threonine, tryptophan, tyrosine, and phenylalanine

What enzyme requires selenium (Se) to function?

Glutathione peroxidase

What apoprotein mediates the uptake of remnants by the liver?

Apoprotein E

How many kcal/g are produced from the degradation of fat? CHO? Protein?

9 kcal/g from fat metabolism; 4 kcal/g from both CHO and protein metabolisms

What is the only fatty acid that is gluconeogenic?

Propionic acid

In a diabetic patient, glucose is converted by aldose reductase to what?

Sorbitol (resulting in cataracts)

What apoprotein is produced by the intestinal epithelium?

Apoprotein B-48

How many ATPs/glucose are generated in glycolysis?

Eight ATPs if aerobic, two ATPs if anaerobic (6 ATPs + 2 ATPs + 2 ATPs - 2 ATPs = 8 ATPs)

What enzyme catalyzes the rate-limiting step in heme synthesis?

Ô-aminolevulinic acid (ALA) synthase

Biochemistry I 17

What enzyme deficiency causes cataracts, galactosemia, and galactosuiia?

Galactokinase deficiency

What factors are needed for elongation in prokaryotes?

Elongation factor (EF)-Tu or EF-ts and GTP

What enzyme catalyzes the rate-limiting step in purine synthesis?

PRPP aminotransferase

What enzyme has a 5'-3' synthesis activity and a 3' exonuclease activity?

DNA polymerase III

At what three sites can the HMP shunt enter into glycolysis?

1. Fnictose-6-phosphate 2. Glucose-6-phosphate 3. Glyceraldehyde-3-phosphate

What toxin adenosine diphosphate (ADP) ribosylates via Gs protein to increase cyclic adenosine monophosphate (cAMP)?

Cholera toxin

What two sugars can be used to produce cerebrosides?

1. Glucose 2. Galactose

What are the three exceptions to the rule of codominant gene expression?

Barr bodies in females, T-cell receptor loci, and immunoglobulin light- and heavy-chain loci

What enzyme catalyzes the rate-limiting step in gluconeogenesis?

Fructose-l,6-bisphosphatase

In eukaryotes, what transcription factor binds to the TATA box before RNA polymerase II can bind?

TFIID

What enzyme makes hnRNA/mRNA?

RNA polymerase II

I 18 USMLE Step I Recall

On a Lineweaver-Burke plot, what type of binding has both plots crossing the y-axis in the same spot?

Competitive, reversible inhibition (V the same; increase in K )

What disease presents with weakness and cramps on exercise without an increase in blood lactate levels?

McArdle's disease (muscle glycogen Phosphorylase deficiency)

What direction does RNA polymerase move along the template strand of DNA during transcription?

3'-5' direction, synthesizing RNA in the 5'-3' direction

What glycolytic enzyme has a high V , high K , and

Glucokinase

©

max'

Ç»

is

in'

low affinity for glucose? What five cofactors and coenzymes are required by pyruvate dehydrogenase?

1. 2. 3. 4. 5.

What technique uses DNA for analysis?

Southern blotting

What is the end-product(s) of: Fatty acid synthesis? Fatty acid oxidation?

What pattern of genetic transmission is characterized by the following: both males (M) and females (F) are affected, M-to-M transmission may be present, there is at least one affected parent in each generation, and only one mutant allele is needed for the disease to be present?

Ribothymidine triphosphate (TTP) Lipoic acid Coenzyme A from pantothenate NAD(H) (from niacin or tryptophan) FADH, (from riboflavin)

Palmitate Acetyl CoA and propionyl CoA (in oddchain fatty acids) Autosomal dominant

Biochemistry I I 9

What is the primary screening test used to detect HrVinfected individuals? The primary confirmatory test?

The enzyme-linked immunosorbent assay (ELISA) is used as a screening test because it is very sensitive; the Western blot is used as a confirmatory test because it detects antibodies (protein) to the HIV virus.

What cytoplasmic pathway functions to produce NADPH and be a source of ribose 5-phosphate?

Hexose monophosphate (HMP) shunt

How many high-energy bonds does the cycle of elongation cost?

Four high energy bonds, two from ATP in AA activation and two from guanosine triphosphate (GTP)

What AA is a precursor of the following substances: Serotonin?

Tryptophan

y-Aminobutyric acid (GABA)?

Glutamate

Histamine?

Histidine

Creatine?

Glvcine/arginine

Nicotinamide adenine dinucleotide (NAD)?

Tryptophan

N 2 0?

Arginine

What substrate builds up in Tay-Sachs disease?

Ganglioside GM,

What are the three stages of protein synthesis?

Initiation, elongation, and termination

What enzyme is used to remove the hairpin loop when producing cDNA from mRNA?

SI nuclease

Chapter 4

Microbiology and Immunology

MICROBIOLOGY What are the components of the rubella triad?

1. Patent ductus arteriosus (PDA) 2. Cataracts 3. Mental retardation

What three organs can be affected by Trypanosoma

1. Heart 2. Esophagus 3. Colon Remember, you get "megas": 1. Cardiomegaly 2. Megaesophagus 3. Megacolon

What are the three naked DNA viruses?

1. Parvovirus 2. Adenovirus 3. Papova\irus (PAP)

Which organism causes epidemic typhus?

Rickettsia prowazekii (tmnk to periphery rash)

What antibody is an indication of recurrent disease for hepatitis?

Hepatitis Be antibody

What three bacteria are quellung reactive test positive?

1. Neisseria meningitidis 2. Haemophilus influenzae 3. Streptococcus pneumoniae

What two viruses cause pancreatitis?

1. Mumps 2. Coxsackie B4Ü

Which organism causes San Joaquin fever?

Coccidioides immitis

With what virus do you see Koplik's spots and WarthinFinkeldey cells, and possibly

Rubeola (measles)

120

Microbiology and Immunology 121

subacute sclerosing panencephalitis? With what two virases are Reye's syndrome associated?

Varicella virus Influenza virus

What bacteria get inoculated into the body by a puncture wound in the skin and also inhibit glycine and y-aminobutyric acid (GABA)?

Clostridium tetani

What three carcinomas are associated with EBV?

1. Burkitt's lymphoma 2. Nasopharyngeal 3. Thymic

What is the only virus to carry its own ribosomes?

Arenavirus

What organism is associated with gastritis and ulcers?

Helicobacter pylori

What is the only dsDNA virus?

Reovirus

What two viruses do not get their envelope from budding but actually from coding?

HIV and poxvirus

What is the direction of the strand if a viras has infectious + RNA?

5 - 3 ' RNA

Which organism causes Lyme disease?

Borrelia burgdorferi

What is die leading cause of diarrhea in die United States?

Campylobacter jejuni

What virus is associated with Negri bodies?

Rabies vims

What antigen is needed to diagnose an infectious patient with hepatitis B?

Hepatitis Be antigen

122 USMLE Step I Recall

Rice water stools are indicative of what organism?

Vibrio cholerae

What is the reservoir for the toga virus?

Birds

What organism needs factor X and nicotinamide adenine dinucleotide (NAD) in order to grow on growth medium?

Haemophilus influenzae type B

To what family of viruses do dengue, St. Louis, and yellow fever belong?

Flavivirus

What infective bacteria are found in undercooked hamburgers?

Escherichia coli strain 0157H7ss

What type of vaccine is the measles, mumps, and rubella (MMR) vaccine?

Live, attenuated vaccine

What virus is responsible for causing the croup and also the common cold in the young and the old?

Parainfluenza virus

What virus causes hoofand-mouth disease?

Vesicular stomatitis vims

What virus is associated with erythema infectiosum or fifth disease?

Parvovirus B-19

Which organism causes multiple infections by antigen switching?

Borrelia recurrentis

What organism causes Q fever?

Coxiella burnetii

What two genera are spore formers?

1. Clostridia 2. Bacillus

Microbiology and Immunology 123

What fungus is seen as colored cauliflower lesions?

Chromomycosis

What organism causes kala-azar, which is associated with hyperpigmentation of the skin, enlargement of the spleen, and decreased bone marrow activity?

Leishmania donovani

What is the only herpes virus to cross the placenta?

CMV

What species of bacteria is associated with whooping cough?

Bordetella pertussis

In what trimester is the fetus most vulnerable to congenital rubella syndrome?

The first trimester

What are the only two viruses where naked dsDNA is NOT infectious?

1. Poxvirus 2. Hepatitis B vims

What fungus is stained positive with India ink?

Cryptococcus neoformans

What fungus is found in pigeon droppings?

Cryptococcus neoformans

What is the most common cause of urinary tract infections?

Escherichia coli

With what organism is "currant jelly" sputum associated?

Klebsiella pneumoniae

Which hepatitis virus is an RNA viroid-Iike viras that needs hepatitis B to be infective?

Hepatitis D

124 USMLE Step I Recall

What fungus is associated with rose gardener's disease?

Sporothrix schencki,

What does Candida albicans do that distinguishes it from other fungi?

It forms a germinal tube at 37°C.

What is the causative agent of orchitis, parotitis, and pancreatitis?

Mumps vims

What are the only two picornavirases that do NOT lead to aseptic meningitis?

1. Rhinovirus 2. Hepatitis A vims

With what organism are intracellular Gram-negative inclusions in neutrophüs associated?

Neisseria gonorrhoeae

What bacteria are found in poorly preserved canned food and cause flaccid paralysis?

Clostridium botulinum

Which three organisms cause heterophUic negative mononucleosis?

1. CMV 2. Toxoplasma gondii 3. Listeria

What are the three nonenveloped RNA viruses?

1. Picornavirus 2. Calicivims 3. Reovirus (PCR)

Antibodies to what hepatitis B antigen provide immunity?

Antibodies to hepatitis Bs antigen

What virus is associated with Guarnieri boches?

Vuiola

What is the only Plasmodium that is quartan?

Plasmodium malariae; the others are tertian.

Microbiology and Immunology 125

What virus affects the motor neurons in the anterior horn?

Poliovirus

Which two organisms can cause sulfur granules in the pus?

1. Actinomyces 2. Nocardia

What people are "protected" from malaria?

People with heterogenous sickle cell trait

What is the most common cause of pneumonia in young children?

Mycopla

What bacteria cause pseudomembranous colitis?

Clostridium difficile

What glycoprotein in the HIV viras attaches to CD4?

GP120

What virus binds to: CD4?

HIV

ß2-Microglobulin?

CMV

Complement factor C3?

Epstein-Barr vims (EBV)

Acetylcholine (ACh) receptors?

Rabies vims

What bacteria are responsible for food poisoning from rice, fried rice, and reheated foods?

Bacillus cereus

What type of Plasmodium is banana or crescent shaped when stained with Giemsa stain?

Plasmodium falciparum

Which hepatitis vinas is an enveloped RNA flavivirus, which is known for postinfusional hepatitis?

Hepatitis C

126 USMLE Step I Recall

What type of hepatitis has the highest mortality rate among pregnant women?

Hepatitis E

Which type of hepatitis is a Picornavirus?

Hepatitis A (infectious)

What is the first antigen seen in an individual with hepatitis?

Hepatitis Bs antigen (incubation period)

In the window phase of a hepatitis B infection, which antibodies do you see?

Hepatitis Be and e antibodies. You see the antibodies c and e.

What bacteria are associated with food poisoning from ham, potato salad, and custards?

Staphylococcus aureus

What types of Plasmodium produce latent hypnozoites in the liver, which can cause a relapse?

Plasmodium vivax and Plasmodium ovale

Which organism has protein A for an antiopsonization defense?

Staphylococcus aureus

Which agent causes pneumonia in college students and military recruits?

Mycoplasma pneumoniae

What organism stains bipolar and causes buboes?

Yersinia pestis

What virus is associated with heterophil-positive mononucleosis?

EBV

What organism would cause a patient to present with constant dian-hea after drinking mountain stream water on a camping trip?

Giardia lamblia

Microbiology and Immunology 127

Which type of hepatitis is a calicivirus?

Hepatitis E (enteric)

What virus is helical and has HN and F glycoprotein spikes?

Paramyxovirus

Which spirochete causes Rocky Mountain spotted fever?

Rickettsia rickettsU (wrist to trunk rash)

What bacteria are responsible for woolsorters' disease?

Bacillus anthracis

What fungus is found in soil with bird or bat feces?

Histoplasma capsulatum

Which organism causes trench mouth?

Fusobacteriui

Which virus is associated with hairy T-cell leukemia?

Human T-cell leukemia virus types I and II (HTLV-I and HTLV-II)

What is the most common cause of pneumonia in children 1 year old or younger?

Respiratory syncytial vims (RSV)

What are the four segmented RNA viruses?

1. Bunyavirus 2. Orthomyxovirus 3. Reovirus 4. Arenavirus (BORA)

What bacteria cause subacute endocarditis and dental caries?

Streptococcus viridans

What is the only diploid virus?

Retrovirus

Which organism causes trench fever?

Rochalimaea quintana

What glycoprotein (GP) in the HIV virus is used for fusion?

GP41

128 USMLE Step I Recall

Which organism causes Wed's disease?

Leptospira

What yeast is urease positive?

Cryptococcus neoformans

What two viruses have neuraminidase activity?

1. Influenza 2. Mumps

What fungus is seen as a yeast with broad-based buds and a double refractile cell wall?

Blastomyces dermatitidis

What is the tenn for the period from the onset of an infection to the appearance of the virus extracellularly?

Latent period

What is the only DNA viras that has the reverse transcriptase enzyme?

Hepadnavinis

What type of Plasmodium affects: Only mature red blood cells (RBCs)?

Plasmodia m malariae

Only reticulocytes?

Plasmodium vivax

RBCs of all ages?

Plasmodium falciparum

What two bacteria are associated with drinking unpasteurized milk?

1. Brucella 2. Listeria (has tumbling motility)

What two antigens need to be positive for a patient to have chronic active hepatitis?

1. Hepatitis Bs 2. Hepatitis e antigen

What virus is associated with intranuclear inclusions known as "owl's eye" inclusions?

Cytomegalovirus (CMV)

Microbiology and Immunology 129

What bacteria constitute the most common cause of nosocomial infections in b u m patients and in patients with cystic fibrosis?

Pseudomonas

What is the most common cause of bacterial pneumonia?

Streptococcus pneumoniae

What are the three C's of measles?

1. Cough 2. Coryza 3. Conjunctivitis

What is the most common cause of meningitis in: Children younger than 3 months of age?

Streptococcus agalactiae or Escherichia colt

Nonimmunized children 12 months to 6 years old?

Haemophilus influenzae type b

Immunized children 12 months to 6 years old?

Streptococcus pneumoniae

Military recruits?

Neisseria meningitidis

HIV-(-/immunocompromised persons?

Cryptococcus neoformans

Adults?

Streptococcus pneumoniae

Which organism, transmitted by sexual contact, is almost diagnostic by the foul-smelling, green discharge from the vagina and its associated itch?

Trichomonas vaginalis

What vector is associated with malaria?

Anopheles mosquito

What is the only ssDNA virus?

Parvovirus

130 USMLE Step I Recall

IMMUNOLOGY What cell type recognizes MHC class I?

Cytotoxic T cell (CD8)

What cytokine stimulates stem cell differentiation?

IL-3

Which cytokine is chemotactic for neutrophils?

IL-8

What immunoglobulin activates the alternate pathway, neutralizes bacterial endotoxins and viruses, and prevents bacterial adherence?

IgA

What is the only IgG that cannot bind to Staphylococcus protein A?

IgG3

What cell surface marker is found on activated helper T cells?

CD40

What immunoglobulin is the first antibody in an immune response?

IgM

Which complement fragments cause lysis of cells?

C5b-9

What aspect of the complement system is deficient if there are repeated gonococcal infections and recurrent episodes of meningococcal meningitis?

C5, 6, 7, or 8

What are the five main oxidizing reactions that are used to kill ingested organisms?

1. H 2 0 2 2. Superoxide 3. Hydroxyl radical 4. Myeloperoxidase 5. Hypochlorous acid

Microbiology and Immunology 131

What cell surface marker is used to distinguish different stages in the maturation of T cells?

CD3

Which cell surface marker binds to C3d fragments?

CD21

Which chromosome is associated with major histocompatibility complex (MHC) genes?

Chromosome 6

What cell surface marker is found on blood B cells?

CD19

What immunoglobulins are found in an infant at birth?

Maternal IgG and fetal IgM

What are defined by antigen-binding specificity?

Idiotypes

What immunoglobulin is responsible for antibodydependent cell-mediated cytotoxicity (ADCC) of parasites, has a high-affinity Fc receptor on mast cells and basophils, and is responsible for the allergic response?

IgE

What cytokine, produced by stromal cells of the bone marrow, is important in lymphoid development?

IL-7. A "7" upside down looks like an "L" (L for Lymphoid).

What are the three rules of clonal selection?

1. One cell type 2. One antibody type 3. Random selection of hypervariable regions, and only cells with bound antigen undergo clonal expansion

What is the valence of an immunoglobulin molecule equal to?

The number of antigens that the antibody can bind

132 USMLE Step I Recall

What are different classes and subclasses of the same gene products known as?

Isotypes

When is the last time that maternal IgG is seen in circulation?

Between 9 and 15 months

What cytokines are secreted by helper T cells to activate the antigen-presenting cells (APCs)?

Interferon (INF)-yand IL-4

With what area of the spleen are the T cells associated?

Periarteriolar lymphatic sheath (PALS)

What protein is used to differentiate MHC class I from MHC class II, and on what chromosome is it found?

ß,-Microglobulin, on chromosome 15

At what stage of the immune response do you see an increase in serum-specific antibody levels?

Log phi

Which immunoglobulin is found as a pentamer and activates complement?

IgM

Which IgG cannot activate complement?

IgG4

What substance is secreted by activated helper T cells to induce T- and B-cell division?

Interleukin (IL)-2

What type of cell can never leave the lymph node?

Plasma cell

What type of cell does an antigen-stimulated B cell turn into if there is a continuous supply of antigen?

Plasma cell

Microbiology and Immunology 133

What is the tenn for a single isolated antigenic detenninant?

Hapten (not immunogenic)

What costimulatory molecules are necessary for effective T-cell and B-cell signaling? What immunoglobulin is responsible for activation of complement, opsonization, and ADCC, and is actively transported across the placenta?

IgG

What subdivision of MHC is found on all nucleated cells?

MHC class I (three subtypes: -A, -B, -C)

Which four helper T-cell cytokines are involved in differentiation?

1. IL-4 2. IL-5 3. IL-6 4. IL-10

Which complement fragment is deficient if a patient presents with repeated infections, fever, rash, and arthralgia?

C3

What are the four chemotactic agents?

1. 2. 3. 4.

Which protein prevents internal binding of selfproteins within an MHC class II cell?

Invariant chain

What cytokine, produced by stromal cells of the bone marrow, is important in myeloid development?

IL-3. A "3' on its side looks like an "nï ; "m" for myeloid.

C5a Leukotriene B4 IL-8 Bacterial peptides

3 ^

134 USMLE Step I Recall

By which process do antibodies make microorganisms more easily ingested via phagocytosis?

Opsonization

Which process is involved in rearranging one heavy-chain gene to produce a functional gene product while it shuts off the rearrangement and expression of the other alleles to ensure that one antibody type is made?

Allelic exclusion

What is the tetrad of JarischHerxheimer reaction?

Rigors, leukopenia, decrease in blood pressure, and increase in temperature

What three factors cause opsonization?

IgG, C3b, and mannose-binding protein

What cell surface marker do all T cells have?

CD3

What is the major antibody of internal secretions (blood, cerebrospinal fluid, lymph)?

IgG

Which region of the variable domain comprises the antigen-binding site of the antibody?

Hypervariable region (three per light chain; three per heavy chain)

What would be the result if an antibody were cleaved with pepsin?

There would be a Fab' region; thus, it would still be able to participate in precipitation and agglutination.

What type(s) of immune response is the body capable of making when presented with a killed vaccine?

Humoral only

What is the major antibody of external secretions?

IgA

Microbiology and Immunology 135

What is the main cell type of chronic inflammation?

Macrophages

What three complement fragments are also anaphylatoxins?

1. C3a 2. C4a 3. C5a

What region of the immunoglobulin does not change with class switching? What immunoglobulin is a marker for mature B cells and is the antigen receptor for B cells?

Hypervariable region

IgD

What stage of the immune response is involved from the time when we are first presented with an antigen to the first time that there are detectable levels of antibody in the serum?

Lag phase

What are the genetic variants of a molecule within members of the same species?

Allotypes

What type(s) of immune response is the body capable of making when presented with a live vaccine?

Humoral and cell mediated

A deficiency in C I esterase inhibitor (Cl-INH) results in what disease?

Hereditaiy angioedema

Copier S

Physiology

What phase of the menstrual cycle is dominated by estrogen? Progesterone?

The follicular phase is estrogen dependent with increased FSH levels, while the luteal phase is progesterone dependent.

What area of the GI tract has the highest activity of brush border enzymes?

jejunum (upper)

Under resting conditions, what is the main determinant of cerebral blood flow?

Arterial PCO, levels are proportional to cerebral blood flow.

How does ventricular repolarization take place (base to apex or vice versa)?

Repolarization is from base to apex and from epicardium to endocardium.

What determines the overall force generated by the ventricular muscle during systole?

The number of cross-bridges cycling during contraction—the greater the number, the greater the force of contraction

What four characteristics are common to all proteinmediated transportation?

1. 2. 3. 4.

What are the three characteristics of an action potential?

1. All or none 2. Propagated 3. No summation

What region of the lung has the greatest blood flow?

The base

In the systemic circulation, what blood vessels have the largest pressure drop? Smallest pressure drop?

Arterioles have the largest pressure drop, whereas the vena cava has the smallest pressure drop in systemic circulation.

136

More rapid than diffusion Zero-order kinetics Chemical specificity Competition for carriers

Physiology 137

What type of circuit is being described when the total resistance is always greater than the sums of the individual resistors?

Series circuit

What is the rate-limiting step in the formation of büe acids?

7ct-Hydroxylase

What lecithin:sphingomyelin ratio indicates lung maturity?

Lecithimsphingomyelin ratio of 2.0 or greater

Up to how many hours postejaculation do sperm retain their ability to fertilize the ovum?

Up to 72 hours; the ovum loses its ability to be fertilized anywhere from 8 to 25 hours after release.

What three organs are responsible for peripheral conversion of thyroxine (T4) to triiodothyronine (T3)?

Liver, kidneys, and pituitary gland via 5' deiodinase enzyme

Name the phase of the ventricular muscle action potential based on the following information: Slow channels open allowing calcium influx, voltage-gated potassium channels are closed, there is potassium efflux through ungated channels, and it is the plateau stage

Phase 2

There is slight repolarization secondary to potassium and the closure of the sodium channels

Phase 1

Fast channels open then quickly close and sodium influx results in depolarization

Phase 0

138 USMLE Step I Recall

Slow channels close, voltage-gated potassium channels reopen with a large influx of potassium, and the cell quickly repolarizes

Phase 3

What is the ratio of pulmonary to systemic blood flow?

1:1. Remember, the flow through the pulnionaiy circuit and the systemic circuit are equal.

The force the ventricular muscle needs to generate to expel the blood into the aorta is known as?

Afterload

Name the following muscle type based on the following histologic features: Actin and myosin in sarcomeres; striated; uninuclear; gap junctions; troponin-calcium binding complex; T tubules and the sarcoplasmic reticulum (SR) fonn dyadic contacts; voltagegated calcium channels

Cardiac muscle

Actin and myosin in sarcomeres; striated; multinuclear; lacks gap junctions; troponin- calcium binding; T tubules and SR form triadic contacts; highest adenosine triphosphatase (ATPase) activity; no calcium channels

Skeletal muscle

Actin and myosin not in sarcomeres; nonstriated; uninuclear; gap junctions; cahnodulin-calcium binding; lacks T tubules, voltage-gated calcium channels

Smooth muscle

Physiology 139

What two pituitaiy hormones are produced by acidophils?

GH and prolactin are produced by acidophils; all others are by basophils.

What happens to capillary oncotic pressure with dehydration?

Oncotic pressure increases because of the removal of water.

What are the four ways to increase total peripheral resistance (TPR)?

1. 2. 3. 4.

How does ventricular depolarization take place (base to apex or vice versa)?

Depolarization is from apex to base and from endocardium to epicardium.

Which region in the lungs gives the best ventilation/ perfusion ratio?

The hilum

What are the four characteristics of all proteinmediated transportation?

1. Competition for carrier with similar chemical substances 2. Chemical specificity needed for transportation 3. Zero-order saturation kinetics (transportation is maximal when all transporters are saturated) 4. Rate of transportation is faster than if by simple diffusion

What cell converts androgens to estrogens? What hormone acts on

Granulosa cell

Decrease the radius of the vessel Increase the length of the vessel Increase the viscosity Decrease the number of parallel channels

FSH

this cell? What hormone causes contractions of smooth muscle, regulates interdigestive motility, and prepares the intestine for the next meal?

Motilin

140 USMLE Step I Recall

How is velocity related to the total cross-sectional area of a blood vessel?

Velocity is inversely related to cross-sectional area.

Is excretion greater than or less than filtration in order for net secretion to occur?

Excretion is greater than filtration for net secretion to occur.

What is the thin filament that binds to calcium?

Troponin

Do the PO„ peripheral chemoreceptors of the carotid body contribute to the normal drive for ventilation?

Under normal resting conditions no, but they are strongly stimulated when PO, arterial levels decrease to 50 to 60 mm Hg, resulting in increased ventilatory chive

What is the role of the negative charge on the filtering membrane of the glomerular capillaries?

The negative charge inhibits the filtration of protein anions.

Where does the inherent rhythm for respiration originate?

In the medullary center of the medullary oblongata

Does the left or right vagus nerve innervate the SA node?

The right vagus innervates the SA node and die left vagus innervates die AV node.

What cells of the thyroid gland are stimulated in response to hypercalcemia?

The parafollicular cells of the thyroid (C cells) release calcitonin in response to hypercalcemia.

What is the most important factor in determining the resistance of a vessel?

The radius; resistance is inversely proportional to the fourth power of the radius (the law of Laplace).

Is most of the coronary artery blood flow during systole or diastole?

Diastole. During systole the left ventricle contracts, resulting in intramyocardial vessel compression, and therefore very little blood flow in the coronary circulation.

Physiology 141

During inspiration, which region of the lung receives the greatest level of ventilation: the apex or the base?

The base

True or false—the blood stored in the systemic veins and the pulmonary circuit are considered part of the cardiac output.

False; cardiac output refers to circulating blood volume. The blood in the systemic veins and the pulnionaiy circuits are storage reserves and therefore are not considered in cardiac output.

What acid-base disturbance is produced from vomiting?

Hypokalemic metabolic alkalosis occurs from vomiting because of the loss

ofH + , K+, andCl". How many days prior to ovulation does an LH surge occur in the menstrual cycle? What component of an electrocardiogram (ECG) is associated with the following: Due to the conduction delay in the AV node?

1 day prior to ovulation

PR interval

Ventricular depolarization?

QRS complex

Atrial depolarization?

P wave

Ventricular repolarization?

T wave

The amount of air that enters or leaves the respiratoiy system in a single respiratory cycle is known as what?

Tidal volume

What pancreatic islet cell secretes glucagons?

Alpha cells; glucagon has stimulatory effects on beta cells and inhibitory effects on delta cells.

142 USMLE Step I Recall

With a decrease in arterial diastolic pressure, what happens to: Stroke volume?

Decreases

TPR?

Decreases

Heart rate?

Decreases

What is the only way to increase 0 2 delivery in the coronary circulation?

Increasing blood flow is the only way to increase O, delivery in the coronaiy circulation because extraction is nearly maximal during resting conditions.

What causes peripheral chemoreceptors to be stimulated?

A decrease in the arterial PO„ H + , and PCO, (not the normal drive for ventilation)

The repolarization phase of the action potential is caused by what?

K+ efflux (Depolarization opens the gates.)

Relaxation of the lower esophageal sphincter during swallowing is under the control of what hormone?

Vasoactive intestinal peptide (VIP) is an inhibitory parasympathetic neurotransmitter that results in relaxation of the lower esophageal sphincter.

What does a Zoss of afferent activity from the carotid sinus onto the medulla signal?

A loss of afferent activity indicates a decrease in BP; an increase in afferent activity indicates an increase in BP.

What cells of the parathyroid gland are simulated in response to hypocalcemia?

The chief cells of the parathyroid gland release PTH in response to hypocalcemia.

What component of the ANS is responsible for movement of semen from the epididymis to the ejaculatory ducts?

Sympathetic neivous system

Physiology 143

What organ of the body has the smallest arteriovenous (AV) oxygen difference?

The renal circulation has the smallest AV O., (high venous PO,) difference in the body due to the overperfusion (because of filtration) of the kidneys.

The clearance of what substance is the gold standard of RPF?

Para-aminohippurate (PAH)

What hormone thins cervical mucus, stimulates LH receptors on granulosa cells, elicits the LH surge, and increases proliferation of the uterine mucosal layers?

Estradiol

What thyroid enzyme is needed for oxidation of

I- to r?

Peroxidase; it is also needed for iodination and coupling inside the follicular cell.

Which CHO is independently absorbed from the small intestine?

Fructose; both glucose and galactose are actively absorbed via secondary active transport.

What is the nontitratable acid that buffers secreted H + in the kidney buffered as?

NH 4 + (ammonium). H,P0 4 (dihydrogen phosphate) is the titratable acid that buffers secreted H + .

What is the best measure of total body vitamin D, if you suspect a deficiency?

Serum 25-hydroxy-vitamin D (25-OH-D)

What are effects of parathyroid hormone (PTH) in the kidney?

PTH increases Ca 2+ reabsorption in the distal convoluted tubule (DCT) of the kidney and decreases P0 4 ~ reabsorption in the PCT.

What four conditions result in secondary hyperaldosteronism?

1. 2. 3. 4.

Congestive heart failure Vena caval obstruction/constriction Hepatic cirrhosis Renal arteiy stenosis

144 USMLE Step I Recall

Why is there an increase in filtration fraction (FF) if the GFR is decreased under sympathetic stimulation?

Because renal plasma flow (RPF) is markedly decreased while GFR is only minimally diminished, resulting in an increase in FF (remember, FF = GFR/RPF)

What is the rate-limiting step in the synthetic pathway of norepinephrine (NE) at the adrenergic nerve terminal?

The conversion of tyrosine to dopamine in the cytoplasm

What is the ratio of T,:T3 secretion from the thyroid

20:1 T4:T3 ratio; there is an increase in the production of T 3 when iodine becomes deficient.

gland?

9

True or false—the secretion of both Cortisol and adrenal androgens are under the control of ACTH.

True

What cell in the heart has the highest rate of automaticity?

Sinoatrial (SA) node. It is the reason why it is the primary pacemaker of the heart.

What is the name of the period in which, no matter how strong the stimulus, a second action potential cannot be generated?

Absolute refractory period

What two organs have local metabolites as the main detenninant of blood flow?

1. Brain (cerebral circulation) 2. Gastrointestinal (GI) tract (after a meal) All other organs are under neural control.

What is the major hormone produced in the following areas of the adrenal cortex: Zona glomerulosa? Zona fasciculata?

Aldosterone Cortisol

Physiology 145

Zona reticularis?

Dehydroepiandrosterone (DHEA; androgens) When going from the outer cortex to the inner layer, remember: "Salt, Sugar, Sex.' The adrenal cortex gets "sweeter" as you go deeper.

How does myelination affect conduction velocity of an action potential?

The greater the myelination is, the greater the conduction velocity.

The clearance of what substance is the goZiZ standard of GFR?

Inulin

What enzyme is needed to activate the following reactions: Trypsinogen to trypsin?

3 ^

Enterokinase

Chymotrypsinogen to ehymotrypsin?

Trypsin

Procarboxypeptidase to carboxypeptidase?

Trypsin

What is the term to describe how easily a vessel stretches?

Compliance (Think of it as distensibility)

What three substances stimulate parietal cells?

ACh, histamine, and gastrin

What gas has a high driving force and low solubility?

O,

The opening of what valve indicates the beginning of the ejection phase of the cardiac cycle?

Opening of the aortic valve terminates the isovolumetric phase and the beginning of the ejection phase of the cardiac cycle.

Which condition involves elevated ACTH and Cortisol levels?

Cushing's disease (pituitary tumor)

146 USMLE Step I Recall

What four factors shift the Hgb—O,, dissociation curve to the right? What is the consequence of this shift?

Increased C0 2 , H + , temperature, and 2,3-bisphosphoglycerate (2,3-BPG) levels all shift the curve to the right, therefore making the 0 2 easier to remove (decreased affinity) from the Hgb molecule.

The closure of what valve indicates the beginning of isovolumetric contraction?

Mitral valve closure indicates the termination of the ventricular filling phase and beginning of isovolumetric contraction.

What form of renal tubular reabsorption is characterized by low back-leaks, high affinity of a substance, easily saturated, and surmised as the entire filtered load is reabsorbed until the carriers are saturated, and then the rest is excreted?

A transport maximum (Tm) system

What two "stress" hormones are under the permissive action of Cortisol?

Glucagon and epinephrine

Which three organs or structures have gluconeogenic capabilities?

1. Liver 2. Kidney 3. GI epithelium

What two factors determine the clearance of a substance?

The plasma concentration and the excretion rate of a substance

The size of the anatomic dead space is roughly equal to what?

The person's weight in pounds

What are the primary neurotransmitters at the following sites: Postganglionic sympathetic neurons?

NE

Chromaffin cells of the adrenal medulla?

Epinephrine

Brainstem cells?

Serotonin

The hypothalamus?

Histamine

All motor neurons, postganglionic parasympathetic neurons?

ACh

Autonomic preganglionic neurons?

ACh

Name the hemoglobin (Hgb)-0 2 binding site based on the following information: Least affinity for O,, requires the highest PO, levels for attachment (approximately 100 mm Hg)

Site 4

Greatest affinity of the three remaining sites for attachment, requires PO„ levels of 26 mm Hg to remain attached

Site 2

Remains attached under most physiologic conditions

Site 1

Requires a P 0 2 level of 40 mm Hg to remain attached

Site 3

What vitamin needs thyroid hormone for conversion to its active form?

Vitamin A

The surge of what hormone induces ovulation?

LH

148 USMLE Step I Recall What are the four major anabolic hormones?

1. 2. 3. 4.

What three organs are necessary for the production of vitamin D 3 (cholecalciferol)?

Skin, liver, and kidneys

What hormone causes contractions of the gallbladder, augments the action of secretin to produce an alkaline pancreatic juice, inhibits gastric emptying, and increases constriction of the pyloric sphincter?

Cholecystokinin (CCK)

What component of the autonomic nervous system (ANS) is responsible for the dilation of the blood vessels in the erectile tissue of the penis, resulting in an erection?

Parasympathetics (Parasympathetics point, sympathetics shoot.)

What is the depolarization phase of an action potential caused by?

Na + influx

What three structures increase the surface area o f t h e G I tract?

1. Plicae circularis (X3) 2. Villi (X30) 3. Microvilli (X600)

What is secretin's pancreatic action?

Secretin stimulates the pancreas to secrete a HCO~-rich solution to neutralize the acidity of the chime entering the duodenum.

What forms of fatty acids are absorbed from the small intestine mucosa by simple diffusion?

Short-chain fatty acids

Insulin Thyroid hormone GH Sex steroids

Physiology 149

What are the five effects insulin has on fat metabolism?

1. Increased glucose uptake by fat cells 2. Increased triglyceride uptake by fat cells 3. Increased conversion of CHOs into fat 4. Decreased lipolysis in fat tissue 5. Decreased ketone body formation

What region of the nephron has the highest osmolality?

Tip of the loop of Henle (1200 mOsm/L)

What are the three main functions of surfactant?

1. Lowers surface tension, so it decreases recoil and increases compliance 2. Reduces capillary filtration 3. Promotes stability in small alveoli by lowering surface tension

What endocrine abnormality is characterized by the following changes in PTH, Ca 2+ , and inorganic phosphate (Pi): PTH decreased, Ca 2+ increased, Pi increased?

Secondary hypoparathyroidism (vitamin D toxicity)

PTH increased, Ca 2+ decreased, Pi decreased?

Secondaiy hyperparathyroidism (vitamin D deficiency/renal disease)

PTH decreased, Ca 2+ decreased, Pi increased?

Primary hypoparathyroidism

PTH increased, Ca 2+ increased, Pi decreased?

Primary hyperparathyroidism

Serum concentration of what substance is used as a clinical measure of a patient's GFR?

Creatinine

What is the main factor affecting PaC0 2 ?

Alveolar ventilation (Hyperventilation decreases PaCO, and vice versa. Body metabolism also affects PaCO,.)

150 USMLE Step I Recall

What is the central chemoreceptors' main drive for ventilation?

Cerebrospinal fluid (CSF) H + levels, with acidosis being the main central drive, resulting in hyperventilation (the opposite being tme with alkalosis)

Does the heart rate determine the diastolic or systolic interval?

Heart rate determines the diastolic interval and contractility determines the systolic interval.

What primary acid-base disturbance is caused by a decrease in alveolar ventilation (increasing C 0 2 levels) resulting in the reaction shifting to the right and increasing H+ and HCO- levels?

Respiratory acidosis (summary: high C0 2 , high H + , slighdy high H C 0 3 )

What part of the autonomic nervous system is the main controller of blood flow when a person is at rest?

Sympathetic nervous system a Constricts; ß, dilates.

What is the relationship between ventilation and P C 0 2 levels?

They are inversely related. If ventilation increases there will be a decrease in PC0 2 levels, and vice versa.

What does angiotensin II do to restore blood pressure?

It has a direct vasoconstrictive effect.

If the AV difference is "+", is the substance extracted or produced by the organ?

A "+"AV difference indicates that a substance is extracted by the organ, and if—", then it is produced by the organ.

What three lung volumes cannot be measured with a spirometer?

1. Residual volume 2. Total lung capacity 3. Functional residual capacity

What is the main factor determining FF?

Renal plasma flow (decrease flow, increase filtration fraction)

What four factors cause aldosterone to be released ?

1. Conversion of angiotensin I to angiotensin II 2. Hyperkalemia

Physiology 151

3. Hyponatremia 4. A decrease in blood volume What hormone is characterized by the following renal effects: Calcium reabsorption, phosphate excretion?

PTH

Calcium excretion, phosphate excretion?

Calcitriol

Calcium reabsorption, phosphate reabsorption?

Vitamin D 3

Where is the last conducting zone of the lungs?

Terminal bronchioles (No gas exchange occurs here.)

What pathophysiologic disorder is characterized by the following changes in Cortisol and ACTH: Cortisol decreased, ACTH increased?

Primary hypocortisolism (Addison's disease)

Cortisol increased, ACTH increased?

Secondary hypercortisolism (pituitaiy)

Cortisol increased, ACTH decreased?

Primary hypercortisolism

Cortisol decreased, ACTH decreased?

Secondary hypocortisolism (pituitaiy)

What happens to total and alveolar ventilation with: Increased rate of breathing?

With an increased rate of breathing the total ventilation is greater than the alveolar ventilation. Rapid shallow breathing increases dead space ventilation with little change in alveolar ventilation. (This is hypo ventilation.)

152 USMLE Step I Recall

Increased depth of breathing?

With an increased depth of breathing both the total and alveolar ventilation

If the ventilation/perfusion ratio is < 1 , what part of the lung is involved and what physiologic process is occurring?

The base, because flow exceeds delivery

Where is the deep breathing center located?

Apneustic center in the pons

What four factors cause the oxygen-hemoglobin dissociation curve to shift to the right?

1. 2. 3. 4.

How many days prior to ovulation does estradiol peak in the menstrual cycle?

2 days prior to ovulation

What is the effect of hypoventilation on cerebral blood flow?

Hypoventilation results in an increase in PC0 2 levels, and therefore an increase in blood flow.

What type of muscle has myoglobin, low CPK, and low ATPase activity; is aerobic; and is for long-term use?

Red muscle (slow-twitch muscle)

What cells of the genitourinary system produce testosterone in males?

Leydig cells

What is the effect of insuhn on protein storage?

Insulin increases total body stores of protein, fat, and CHOs. When you think insulin, you think storage.

A pH of 1 0 5 fL. What is your diagnosis and how will you monitor his response to treatment?

Subacute combined degeneration of the spinal cord is treated with IM vitamin B 12 injections. If treatment is working you will see an increased reticulocyte count on the peripheral smear in about 5 days.

What is the triad of meningitis?

1. Fever 2. Headache 3. Stiff neckNausea, photophobia, and irritability are also commonly seen in patients with meningitis.

Name the type of hypersensitivity reaction based on the following properties: Circulating antibodyantigen (Ab-Ag) immune complexes get deposited in the tissue resulting in

Type III hypersensitivity (immune complex)

Pathology 205 neutrophil attraction and the release of lysosomal enzymes IgE-mediated release of chemical mediators from basophils and mast cells; need prior exposure to Ag in the past; eosinophils amplify and continue reaction; can be systemic or localized

Tvpe I hypersensitivity (anaphylactic)

IgG or IgM Abs against a specific target cell or tissue; complement dependent or antibodydependant cell-mediated cytotoxicity (ADCC)

Type II hypersensitivity (cytotoxic)

Reaction mediated by sensitized T cells

Type IV hypersensitivity (cell mediated)

A 20-year-old college student presents with fever, graywhite membranes over the tonsils, posterior auricular lymphadenitis, and hepatosplenomegaly. What is your diagnosis? What test do you order to confirm your diagnosis?

EBV infections resulting in infectious mononucleosis can be diagnosed by the monospot test. (Remember, it may be negative in the first week of the illness, so retest if you have a high index of suspicion.)

What organism is associated with hyaline membrane formation and cold agglutinins?

Mycoplasma

What pathway in the coagulation cascade is activated by making contact with foreign substances?

Intrinsic pathway

Which of the following is NOT a risk factor for

Cirrhosis and hemolytic anemia are risk factors for pigmented gallstones.

206 USMLE Step I Recall

cholesterol gallstones: pregnancy, oral contraceptive (OCP) use, being female, hemolytic anemia, cùrhosis, and obesity (may be more than one answer)? What CNS tumor is seen in persons between 40 and 50 years of age; occurs in men more than women; affects the cerebral brainstem; is GFAP positive; has a poor prognosis; and microscopically displays pseudopalisades, increased cellularity, pleomorphism, neovascularization, and necrosis?

Glioblastoma multiforme

A chronic alcohol abuser with a history of cirrhosis and portal HTN presents to the ER with sudden onset of massive, bright red, bloody vomit (hematemesis); what would you first diagnose?

Because of the buzzwords—cirrhosis, portal HTN, and sudden onset, bright red vomit—think esophageal (or gastric) varices, then closely peptic ulcer disease (PUD) bleeds. It's either that or you need endoscopy.

What enzyme level is commonly elevated in sarcoidosis?

Angiotensin-Converting enzyme (ACE)

What mahgnant neoplasm of the skin is associated with keratin pearls?

Squamous cell carcinoma

Name the cerebral vessel associated with the following vascular pathologies: Subarachnoid hemonhage

Berry aneurysm in the circle of Willis

Subdural hemorrhage

Bridging veins draining into the sagittal sinus

Epidural hemoiThage

Middle meningeal arteiy

Pathology 207

What variant of polyarteritis nodosa is associated with bronchial asthma, granulomas, and eosinophilia?

Churg-Strauss syndrome

What female genital tract disorder is characterized by obesity, hirsutism, infertility, amenorrhea, elevated LH and testosterone levels, and low FSH levels?

Polycystic ovary disease (Stein-Leventhal syndrome)

What syndrome, due to an adrenal gland adenoma, produces excess aldosterone resulting in HTN, hypokalemia, and low rennin levels?

Conn's syndrome (primary hyperaldosteronism)

What are the three opsonins?

1. C3b 2. Fc region of IgG 3. Mannose-binding protein

What is the term for an BBC with inorganic ironcontaining granules?

Siderocyte

What disease affects the basal ganglia and the substantia nigra; produces cogwheel rigidity, mask-like faciès, and resting tremors; and reveals Lewy bodies microscopically?

Parkinson's disease

In what syndrome does the patient have angiomatosis; renal cell carcinomas; pheochromocytomas; retinal, cerebellar, medulla, or spinal cord hemangioblastomas; and epidermal cysts?

von Hippel-Lindau syndrome

208 USMLE Step I Recall A Japanese man presents with weight loss, anorexia, early satiety, epigastricabdominal pain, and a palpable left supraclavicular lymph node; on endoscopy you find a large, irregular ulcer with elevated margins on the lesser curvature of the stomach. What is your diagnosis?

Gastric carcinoma

What disease, diagnosed by exclusion, involves bilateral hilar lymphadenopathy and noncaseating granulomas in many organs and is most commonly seen in black women?

Sarcoidosis

What dye is a major cause of transitional cell carcinoma of the urinary bladder?

Naphthalene

What form of lymphoma is diffuse in the lymph node, has an IgM spike, slowly evolves to CLL, is seen in the old, and also has liver, spleen, and bone marrow involvement?

Small cell lymphoma

The presence of both testicular and ovarian tissues within one individual defines what condition?

True hermaphrodite

What disorder is due to a deficiency in the enzyme glucocerebrosidase ?

Gauchers disease

What syndrome results if the enzyme a-1-iduronidase is deficient? If L-iduronate sulfatase is deficient?

Hurler's syndrome and Hunter's syndrome, respectively

Pathology 209

What syndrome occurs when pelvic inflammatory disease ascends to surround the liver capsule in "violinstring" adhesions?

Fitz-Hugh-Curtis syndrome

What tumor is seen in the 2- to 4-year-old age group; does not cross the midline; has immature glomeruli, tubules, and stroma; and metastasizes late to the lungs?

Wilms' tumor

What breast malignancy has tumor cells with a "halo" surrounding the nucleus and is an ulceration of the nipple and areola with crusting, Assuring, and oozing?

Paget's disease of the breast

What leukemia is associated with "four-leaf clover" lymphocytes on peripheral blood smear?

Adult T-cell leukemia

What variant of Hodgkin's lymphoma is widespread with extensive fibrosis and necrosis, occurs in older patients, involves many Reed-Sternberg cells, and has a poor prognosis:

Lymphocyte depletion

Trae or false—pancreatic insufficiency results in vitamin B 12 malabsorption.

True; pancreatic enzymes begin the breakdown of the vitamin B10-R complex in the duodenum.

How many Jones criteria are required for the diagnosis of rheumatic fever (major and/or minor)?

Two major or one major and two minor

What disease is due to a deficiency in the enzyme sphingomyelinase ?

Niemann-Pick disease

210 USMLE Step I Recall Calcium pyrophosphate crystals are associated with what fonn of arthritis?

Pseudogout

Codman's triangle (periosteal elevation) on radiography helps diagnose what malignant bone tumor?

Osteosarcoma

Cell fragments of hemolysis are known as what?

Helmet cells (schistocytes)

What two antibodies are used to diagnose Hashimoto's thyroiditis?

Antithyroglobulin and antimicrosomal antibodies

Which form of emphysema is associated with an Oj-antitrypsin deficiency?

Panacinar

What syndrome has massive proteinuria, hypoalbuminemia, hyperlipidemia, and anasarca as its components?

Nephrotic syndrome

What form of endocarditis do patients with systemic lupus erythematosus (SLE) commonly encounter?

Libman-Sacks endocarditis

What is die term for ischemia to the central nervous system (CNS) causing dissolution of the tissue due to the actions of hydrolytic enzymes?

Liquefactive necrosis

With what pathology is deposition of calcium pyrophosphate in patients older than 50 years of age associated?

Pseudogout

Telescoping of the proximal bowel into the distal

Intussusception

Pathology 21 I

segment presenting as abdominal pain, "currant jelly" stools, and intestinal obstruction is known as? True or false—all of the following are risk factors for breast cancer: early menses, late menopause, prior history of breast cancer, obesity, and multiparity.

False; they are all risk factors for breast cancer except multiparity. ZVuZZ/parity, increasing age, and family history in firstdegree relative are also risk factors. Memorize this list!

Which fonn of melanoma carries the best prognosis?

Lentigo maligna melanoma

What organism is commonly associated with intrauterine device (IUD) infections?

Actinomyces

What variant of Hodgldn's lymphoma occurs least frequentiy, is seen in people younger than 35 years of age, is localized, has an excellent prognosis, and involves mainly lymphocytes and a few Reed-Sternberg cells?

Lymphocyte predominant

What leukemia is associated with a chromosome 9,22 translocation; constitutes 80% of childhood leukemias; and has blasts with periodic acid Schiff (PAS)-positive material and terminal deoxy-transferase (TdT) marker present?

Acute lymphocytic leukemia (ALL)

What are the three causes of a microvesicular fatty change in the liver?

Tetracycline toxicity Reyes syndrome Fatty liver of pregnancy

212 USMLE Step I Recall

The sucrose lysis test and Ham's test are used to diagnose what form of anemia?

Paroxysmal nocturnal hemoglobinuria

Is an anti-hepatitis A virus (anti-HAV) IgG Ab associated with immunization or recent infection?

Anti-HAV IgG Abs are associated with immunization or a prior infection. Anti-HAV IgM is associated with acute or recent infection.

What carcinoma presents with hematuria, flank pain, and a palpable mass?

This is the triad of renal cell carcinoma.

What type of neurofibromatosis is associated with bilateral acoustic schwannomas?

Type 2

What disease has the following characteristics: occurrence in women more than men; involvement of the second or third generation; positive ANAs; joint pain; skin rash in a malar distribution; diffuse proliferate glomerulonephritis (GN); LibmanSacks endocarditis; and neurologic disorders?

SLE MD SOAP N HAIR (malar rash; discoid lesion; serologic; oligoarthritis; ANA positive; pleuritis/pericarditis; neurologic comphcations; hematologic [leukopenia/ thrombocytopenia] ; anticardiolipin antiboches; immunologic; and renal)

What are the three platelet aggregating factors?

1. Adenosine diphosphate (ADP) 2. Prostaglandin 3. TXA,

What slow-growing CNS tumor occurs in 30- to 50-year-old patients who have a protracted history of seizures and has a "fried-egg" cellular appearance in a network of "chicken wire"?

Oligodendroglioma

Pathology 213

What human papillomavirus (HPV) serotypes are associated with increased risk of cervical cancer?

HPV serotypes 16, 18, 31, and 33

What is the treatment for physiologic jaundice of newborns?

Phototherapy

What is the "classic" triad of tuberculosis (TB)?

Fever, night sweats, and hemoptysis

What is the only subtype of Hodgkin's lymphoma that is more commonly seen in females?

Nodular sclerosis

What cerebral lobes are most commonly affected in herpes encephalitis?

Temporal lobes

Hypoglycemia, sweating, hunger, confusion, and increased C-peptide levels are associated with what pancreatic islet cell tumor?

Insulinoma

What AR disease involves a deficiency in glucocerebrosidase, a huge spleen, and engorged phagocytic cells and is associated with chromosome 1?

Gauchers disease

What protein deficiency results in respiratoiy distress syndrome of newborns?

Deficiency in surfactant

What AR disease involves a substitution of valine for glutamic acid at position 6 on the ß chain?

Sickle cell anemia

214 USMLE Step I Recall

What is the name of the tumor when gastric carcinoma spreads to the ovaries?

Kmkenberg's tumor

What X-linked recessive disease involves a decrease in hypoxanthine guanine phosphoribosyl transferase (HGPRT), mental retardation, self-mutilation, choreoathetosis, spasticity, and an increase in uricemia?

Lesch-Nyhan syndrome

What highly undifferentiated aggressive CNS tumor, of primordial neuroglial origin, develops in chUdren and is associated with pseudorosettes?

Primitive neuroectodermal tumors (i.e., medulloblastomas and retinoblastomas)

What skin condition is a localized proliferation of melanocytes presenting as small, oval, light brown macules?

Benign lentigo

Bleeding and cryoglobulin precipitation at low temperatures, headache and confusion due to hyperviscosity, IgM M-protein spike on serum electrophoresis, and Russell bodies are associated with what lymphoma?

Waldenstrom's macroglobulinemia

In what rare lung mahgnancy have 90% of the persons with die disease had an occupational exposure to asbestos?

Malignant mesothelioma

What vitamin deficiency can result in high-output cardiac failure?

Thiamine

Pathology 215

What are the three causes of angiosarcoma of the liver?

1. Vinyl chloride 2. Thorium dioxide 3. Arsenic

What AD renal disorder is associated with mutations of the PKD 1 gene on chromosome 16 and Berry aneurysms in the circle of Willis, and presents in the fifth decade with abdominal masses, flank pain, hematuria, HTN, and renal insufficiency?

Adult polycystic kidney disease

What gene stimulates apoptosis when DNA repair is unable to be done?

p-53

What pathway in die coagulation cascade is activated after tissue injury?

Extrinsic pathway

Homer-Wright pseudorosettes; "onion skinning" of the periosteum on radiography of the femur, pelvis, and tibia; and chromosome 11;22 translocation are associated with what mahgnant neoplasm of the bone?

Evving's sarcoma

What ovarian pathology is associated with Turner's syndrome or malformation of the genitals and comprises 50% of all malignant germ cell tumors?

Dysgerminoma

What two factors of the complement cascade are deficient in the person widi SLE?

C2 and C4

216 USMLE Step I Recall

Transverse bands on the fingernails seen in patients with chronic arsenic poisoning are called?

Mees' lines

What stromal tumor in males is characterized histologically with "crystalloids of Reinke"?

Leydig cell tumor

True or false—Raynaud's phenomenon has no underlying pathology associated with it.

False; the "disease" has no associated patholog)-; the "phenomenon" is arterial insufficiency due to an underlying disease.

What X-linked recessive disorder is due to an abnormality in die dystrophin gene and presents with onset by age 5 with progressive, proximal muscle weakness; calf pseudohypertrophy; and elevated CPK levels?

Duchennes muscular dystrophy (Remember: Becker's is slower in progress, less severe, later in onset, and lacks cardiac involvement.)

Brown tumors, bone pain, deformities, and fractures due to e.vcess parathyroid hormone (PTH) characterizes what bone disorder?

Osteitis fibrosa cystica (von Recklinghausen disease)

What are the three causes for normochromic normocytic anemia with a normal MCV and a low reticulocyte count?

1. Marrow failure 2. Cancer 3. Leukemia

What platelet disorder is characteristically seen in children following a bout of gastroenteritis with bloody diarrhea?

Hemolytic uremic syndrome

What is the triad of fat embolism?

1. Petechiae 2. Hyperactive mental status

Pathology 217 3. Occurs within 24-48 hours of the initial insult (e.g., long bone fracture) Which subset of M E N syndrome is associated with the following: Medullary carcinoma of the thyroid, p h e o c h r o m o cytoma, and mucocutaneous neuromas?

M E N III (or lib)

Medullary carcinoma of die thyroid, pheochromocytoma, and parathyroid adenomas (or hyperplasia)?

M E N Ha (or Sipple's syndrome)

Parathyroid, pancreatic, and pituitaiy gland tumors, and ZollingerEllison syndrome?

M E N I (or Wermer's syndrome)

What factors in the coagulation cascade n e e d factor Ha (activated) to b e c o m e activated?

Factors V and VIII

E d e m a that has lactate dehydrogenase (LDH) < 2 0 0 , protein level < 2 . 5 , and a specific gravity < 1.020 is known as?

Transudative; exudative has the opposite values and also has an elevated cellular content.

What are the three causes of transudate?

1. Congestive heart failure ( C H F ) 2. Cirrhosis 3. Nephrosis

The unidirectional attraction of cells toward a chemical mediator released during inflammation is known as?

Chemotaxis

Panhypopituitarism secondary to ischemic necrosis and hypotension postpartum is known as?

Sheehan's syndrome

218 USMLE Step I Recall

What AR disease has a deficiency in homogentisic oxidase that causes brittle, fibrillated articular cartilage; blue-black pigmentation of collagen; and urine that turns black upon standing?

Alkaptonuria

Degeneration of y-aminobutyric acid (GABA) neurons in the caudate nucleus resulting in atrophy, chorea, dementia, and personality changes characterizes what AD disorder?

Huntington's disease

RBC remnants of nuclear chromatin seen in asplenic patients are known as?

Howell-Jolly bodies

What type of erythema do you see in: Ulcerative colitis?

Erythema nodosum

Rheumatic fever?

Erythema marginatum

Stevens-Johnson syndrome?

Erythema multiforme

What disease is seen in chUdren younger than 5 years of age, is X-linked recessive, and has the following characteristics: cardiac myopathies, calf pseudohypertrophy, lordosis, protuberant bellies, an increase and then a decrease in creatine Phosphokinase (CPK), and death in the second generation

of life?

Duchennes muscular dystrophy

Pathology 219

True or false—gastroesophageal reflux disease (GERD) is a cause of asthma.

Tme; don't forget this in your differential diagnosis in an asthmatic.

What foci of fibrinoid necrosis are surrounded by lymphocytes and macrophages throughout all the layers of the heart?

Aschoff s bodies of rheumatic fever

True or false—all of the following are risk factors of endometrial carcinoma: obesity, diabetes mellitus (DM), HTN, multiparity, early menarche, and late menopause.

False; they are all risk factors for endometrial carcinoma except multiparity. Nulliparity estrogenproducing tumors, and estrogen replacement therapy are also risk factors for endometrial carcinoma.

What AR disorder is due to a deficiency in glycoprotein Ilb-IIIa, resulting in a defect in platelet aggregation?

Glanzmann's syndrome

What form of nephritic syndrome is associated with celiac sprue and HenochSchönlein purpura?

IgA nephropathy

Name the following descriptions associated with bacterial endocarditis: Retinal emboli

Roth spots

Painful, subcutaneous nodules on fingers and toes

Osier's nodes

Painless, hyperemic lesions on the palms and soles

Janeway lesions

The onset of burning epigastric pain 1-3 hours after eating and that is

Duodenal ulcer

220 USMLE Step I Recall relieved by food "classically" describes what type of PUD? Gastric ulcers associated with severely burned or traumatic patients are known as?

Curling's ulcers (diink curling iron = burn)

What disorder is due to a deficiency in tyrosinase?

Albinism

What are the four cardiac abnormalities associated with tetralogy of Fallot?

1. Shifting of the aorta 2. Hypertrophy of the right ventricle 3. Interventricular septal defect 4. Pulmonary stenosis (SHIP)

Name the cancer associated with the following tumor markers (some may have more than one answer): ß-Human chorionic gonadotropin (hCG)

Choriocarcinomas and trophoblastic tumors

Carbohydrate antigen (CA)-125

Ovarian cancer

CA-19.9 and carcinoembryonic antigen (CEA)

Pancreatic cancer

a-Fetoprotein

Hepatoma and nonseminomatous testicular germ cell tumors

Calcitonin

Medullary carcinoma of the thyroid

Prostate-specific antigen (PSA) and prostatic acid phosphatase

Prostate cancer

Placental alkaline phosphatase

Seminomas

CEA

Cancer of die lung, stomach, colon, and breast

Pathology 221

What factors in the coagulation cascade need Ca 2+ to be activated?

Factors II and X

Name at least three causes of metastatic calcification.

Remember the mnemonic: P = (H)per)Parathyroid/Paget's disease A = Addison's disease M = Milk-alkali syndrome/metastatic cancer S = Sarcoidosis M = Multiple myeloma I = Imniobilization/idiopathic D = Vitamin D intoxication T = Tumors "PAM SMIDT"

What disease presents with severe mental retardation, ventricular septal defect (VSD), asymmetric face, microcephaly, and chromosome 5p deletion?

Cri-du-chat

If you order a ventilation/ perfusion (V/Q) scan for suspected pulnionaiy emboli, is the filling defect seen on the ventilation or perfusion side?

Ventilation of a nonperfused lung segment is highly suspicious for pulmonaiy embolism.

Name the type of regeneration (i.e., labile, stable, or permanent) based on the following examples: Epidermis

Labile

Skeletal muscle

Permanent

Pancreas

Stable

CNS neurons

Permanent

Fibroblasts

Stable

222 USMLE Step I Recall

Hematopoietic cells

Labile

Liver

Stable

Smooth muscle

Stable

Cardiac muscle

Permanent

Mucosal epithelium

Labile

Kidney

Stable

Osteoblasts

Stable (In case you forgot, cells that are labile proliferate diroughout life; stable have a low level of proliferation; and permanent, as its name states, do not proliferate.)

What slow-growing primary CNS tumor, more commonly seen in females, is associated with psammoma bodies?

Meningioma

What bone cell has receptors for PTH?

Osteoblasts (Remember, they modulate the function of osteoclasts.)

What estrogen-producing tumor of the female genital tract is characterized by "Call-Exner bodies"?

Granulosa cell tumor of the ovary

Auer rods are most commonly associated with which subtype of AML?

M3 (promyelocytic leukemia)

What fonn of necrosis is caused by immune-mediated vascular damage?

Fibrinoid necrosis

The "tea and toast" diet is classically associated with what cause of megaloblastic anemia?

Folate deficiency (very common in the

elderly)

Pathology 223

What factor gets activated in the intrinsic pathway of the coagulation cascade? Extrinsic pathway?

Factor XII for the intrinsic pathway; factor VII for the extrinsic pathway

What vaginal tumor is seen in children younger than 4 years old and is characterized as "grape-like," polypoid soft tissue growth?

Sarcoma botryoides (embiyonal rhabdomyosarcoma)

Antinuclear antibodies (ANAs), anti-dsDNA, and smooth muscle antigen (Sm Ag) are all used to diagnose what disease?

SLE

What is an elevated, fluidfilled cavity between the layers >0.5 cm?

Bulla

What type of GN has C3, IgG, C l q , and C4 along with subendothelial deposits?

MPGN type I (two-thirds of the MPGN cases)

What commonly encountered overdose presents with headache, tinnitus, respiratory alkalosis and metabohc acidosis, confusion, vomiting, and tachypnea?

ASA (salicylate)

What bronchogenic carcinoma is associated with an elevated level of Ca 2+ , involves keratin pearls, occurs in men more than women, is associated with smoking, occurs in the major bronchi, and is seen in the central areas of the lung?

Squamous cell carcinoma

What form of necrosis is seen in the lower extremity

Gangrenous necrosis

224 USMLE Step I Recall

or the bowel due to vascular occlusion? What CD4 T-cell receptor does HIV bind to?

gpl2()

Hepatomegaly, ascites, and abdominal pain due to hepatic vein thrombosis is known as what syndrome?

Budd-Chiari syndrome

What pathology has pulmonary and renal basement membrane involvement and crescent fonnation and involves type II hypersensitivity?

Goodpasture's syndrome

With what disease do you see IgA deposits in small vessels of the skin and the kidneys?

Henoch-Schönlein purpura

What disease arises from the adrenal medulla, displaces and crosses the midline, metastasizes early, is the most common solid tumor, and is seen in the 2- to 4-year-old age group?

Neuroblastoma

Time or false—pancreatic delta-cell tumors inhibit cholecystokinin (CCK) secretion, leading to gallstones and steatorrhea.

True; delta-cell tumors produce excess somatostatin, which inhibits CCK, gastrin (hypoehlorhydria), and insulin secretion (diabetes).

A 20-year-old female who was recently diagnosed with a sexually transmitted disease presents to the ER with a tender, painful, swollen, and erythematous knee (monoarticular). What organism is the likely culprit?

Neisseria gonorrhea (With history of STD in patient with monoarticular infectious arthritis, think gonococcus.)

Pathology 225

What fonn of sunlight is the most carcinogenic?

Ultraviolet B (UVB) sunlight

What hereditary bone disorder is due to decreased osteoclast function, resulting in thick, sclerotic bones that fracture easily?

Osteopetrosis (Albers-Schönberg disease)

What vasculitis has the following characteristics: has systemic vasculitis affecting small to mediumsize vessels (except the lungs), affects young males, is 30% HBsAg positive, and has perinuclear antineutrophilic cytoplasmic antibody (P-ANCA) and autoantibodies against myeloperoxidase?

Polyarteritis nodosa

A 60-year-old man presents with back pain (compression spinal fracture), hypercalcemia, increased serum protein, Bence-Jones proteinuria, and monoclonal M-spike on serum electrophoresis. What is your diagnosis?

Multiple myeloma

What syndrome is seen in iron-deficient, middle-aged females widi esophageal webs?

Plummer-Vinson syndrome

What syndrome comprises small cell carcinoma of the lung and myasthenia gravis?

Lambert-Eaton syndrome

What is the collapse of the vertebral body due to TB known as?

Pott's disease

226 USMLE Step I Recall RBC fragments are k n o w n as?

Schistocytes

W h a t v a r i a n t of H o d g k i n ' s l y m p h o m a is t h e m o s t c o m m o n t y p e ; involves women more than men; o c c u r s in a d o l e s c e n c e m o r e t h a n in o l d a g e ; affects t h e l o w e r cervical, s u p r a clavicular, a n d m e d i a s t i n a l lymph nodes with broad b a n d s of fibrous tissue a n d " l a c u n a r " cells; a n d h a s a good prognosis?

Nodular sclerosis

W h a t is t h e t e r m for a l a r g e , i m m a t u r e R B C t h a t is spherical, blue-colored, and without a nucleus?

Reticulocyte

W h a t m y e l o i d d i s o r d e r is characterized by increased h e m a t o c r i t , b l o o d viscosity, basophils, a n d eosinophils; i n t e n s e p r u r i t u s ; a n d gastric ulcers due to histamine r e l e a s e from b a s o p h i l s , i n c r e a s e d LAP, a n d p l e t h o r a ?

Polycythemia vera (Remember, polycythemia vera is a risk factor for acute leukemias.)

Gastric ulcers associated with increased intracranial p r e s s u r e a r e k n o w n as?

Cushing's ulcers

W h a t f o r m of h e m o p h ü i a is X-linked recessive a n d d u e t o a deficiency in factor VIII?

Hemophilia A

A d e f i c i e n c y in t h e e n z y m e hexosaminidase A r e s u l t s in what condition?

Tay-Sachs disease

W h a t a r e t h e t h r e e B's of a d u l t polycystic k i d n e y s ?

1. Big 2. Bilateral 3. Berry aneurysm

Pathology 227 What bronchogenic carcinoma occurs most commonly in men and smokers; is found centrally; and can have ectopic production of adrenocorticotropic honnone (ACTH) or antidiuretic honnone (ADH)?

Small cell carcinoma (oat cell)

What sex cell tumor causes precocious puberty, masculinization, gynecomastia in adults, and crystalloids of Reinke?

Leydig cell tumor

What form of GN is characteristically associated with crescent formation?

Rapidly progressive glomerulonephritis (RPGN)

What tumor comprises 40% of all testicular tumors in children?

Teratoma

What disorder is due to a deficiency in the enzyme phenylalanine hydroxylase?

Phenylketonuria (PKU)

What disease affects medium to small arteries, occurs most commonly in young men, is associated with elevated P-ANCA levels to myeloperoxidase, and involves 30% of patients with hepatitis Bs antigen?

Polyarteritis nodosa (PAN) Remember by P in P-ANCA and P in PAN.

Microcytic anemia with basophilic stippling is seen in what metal poisoning?

Lead poisoning

What AD syndrome presents with hamartomatous polyps in the small intestine and

Peutz-Jeghers syndrome

228 USMLE Step I Recall

pigmentation of the lips and oral mucosa? What lymphoma is a diffusely mixed, diffusely large cell that grows rapidly and consists mainly of B cells along with "null" cells?

Diffuse aggressive lymphoma

Two weeks after a throat infection, a mother brings her child to the ER presenting with fever, malaise, HTN, dark-colored urine, and periorbital edema. What is your diagnosis?

Poststreptococcal GN

True or false—psammoma bodies are seen in medullary carcinoma of the thyroid.

False; elevated calcitonin levels are seen in medullary carcinoma of the thyroid. Psammoma bodies are seen in papUlary carcinoma of the thyroid and ovaries, as well as meningiomas.

What is the most common type of necrosis?

Coagulative necrosis

What causes platelets to bind to the collagen of the basement membrane?

von Willebrand's factor (factor VIII)

What term applies when a single vessel receives blood from both ventricles?

Truncus arteriosus

What X-linked recessive disease involves faUure of maturation of pre-B cells with no surface immunoglobulins, leading to recurrent pyogenic infections in infants?

X-linked agammaglobulinemia

What are the four DNA oncogenic viruses?

1. Human papilloma vims (HPV) 2. Epstein-Barr vims (EBV)

Pathology 229

3. Hepatitis B 4. Kaposi's sarcoma What AR disorder of copper metabolism can be characterized by KayserFleischer rings, decreased ceruloplasmin levels, and increased urinary copper excretion and tissue copper levels?

Wilson's disease (Remember, patients commonly present with psychiatric manifestations and movement disorders, but may be asymptomatic as well.)

What nephritic syndrome has effacement of the epithelial foot processes without immune complex deposition?

Minimal change disease

What are the four reasons for hypochromic microcytic anemia with a low MCV?

1. Sideroblastic anemias (i.e., poiphyrin and heme synthesis disorders) 2. Thalassemia 3. Iron deficiency 4. Lead poisoning

What autosomal dominant (AD) syndrome involves 1000 or more edematous polyps, most commonly affects the colorectal area, and is associated with chromosome 5q21?

Familial polyposis coli

True or false—increased leukocyte alkaline phosphatase (LAP) is associated with CML?

False; increased LAP is seen in "stress" reactions and helps differentiate benign conditions from CML (which has low LAP levels).

What fonn of angina is characterized by: Coronary artery vasospasm, symptom occurrence at rest, ST segment elevation (during episode), and "normal"

Prinzmetal variant angina

230 USMLE Step I Recall

(without symptoms) on electrocardiogram (ECG)? Coronary artery luminal narrowing, symptom occurrence during exertion, and ST segment depression on ECG?

Stable angina

Coronary artery nonocclusive thrombus and symptom occurrence with increasing frequency, duration, and intensity and decreasing activity, frequently at rest?

Unstable (crescendo) angina

The inability of the lower esophageal sphincter to relax with swallowing and a "bird beak" barium swallow result defines what disorder?

Achalasia (Think Chagas' disease if it presents in a person from Central or South America.)

What disease involves "cold" skin abscesses due to a defect in neutrophil Chemotaxis and a serum IgE level >2000?

Job's syndrome

A twisting of the bowel around its vascular axis resulting in intestinal obstruction is known as?

Volvulus

Flattened nose, low-set ears, and recessed chin, seen in patients with bilateral renal agenesis, is known as?

Potter's faciès

What is an elevated, fluidfilled cavity between skin layers up to 0.5 cm?

Vesicle (e.g., poison ivy)

Pathology 231

What syndrome has the following components: hypertension, proteinuria, hematuria, azotemia, and oliguria?

Nephritic syndrome

What disease involves a failure of the third and fourth pharyngeal pouches to develop, with a lack of T-cell immunity causing a poor defense response to certain fungal and viral infections and tetany?

Di Georges syndrome

Increased iron deposition resulting in micronodular cirrhosis, CHF, diabetes, and "bronzing" of the skin is known as?

Hemochromatosis

What is the tenn for a RBC that has a peripheral rim of Hgb with a dark, central, Hgb-containing area?

Target eel

What fonn of vasculitis involves the ascending arch and causes an obliterative endarteritis of the vasa vasorum?

Syphilitic

What retrovirus is associated with adult T-cell leukemia?

Human T-cell lymphotropic virus (HTLV-1)

What cellular process defines irreversible cellular injury?

Vacuolation of the mitochondria

Which is associated with black pigmentation of the liver: Dubin-Johnson or Rotor's syndrome?

Both are AR and have conjugated hyperbilirubinemia, but Dubin-Johnson syndrome is differentiated from Rotor's by the black pigmentation of the liver.

232 USMLE Step I Recall

Air in the pleural space is known as?

Pneumothorax

Granulomas at the lung apex in TB are known as?

Simon focus

What are the two most common origins of pulmonary embolism?

1. Deep veins of the legs 2. The prostatic plexus of veins in the pelvis

What AR disease presents with B- and T-cell deficiencies, lymphopenia, an IgA deficiency, cerebellar problems, and spider angiomas?

Ataxia-telangiectasia

Time or false—all of the following are risk factors for cervical cancer: multiple pregnancies, early age of intercourse, multiple sexual partners, OCP use, smoking, HIV, and sexually transmitted diseases (STDs).

True; don't forget this list; you will be asked.

What neuroendocrine tumor produces excess serotonin; is associated with diarrhea, flushing, bronchospasms, and wheezing; and is diagnosed by elevated urinary 5-HIAA levels?

Carcinoid tumors

What enzyme is lacking in the autosomal recessive type of severe combined immunodeficiency?

Adenosine deaminase

What chemical can be potentially dangerous if you work in the aerospace industry or in nuclear plants?

Beryllium

Pathology 233 What is the chromosomal translocation associated with follicular lymphomas?

Chromosome 14; 18 translocation

What type of hemoglobin (Hgb) is increased in patients with sickle cell anemia who take hydroxyurea?

Hgb F

What is the name of the ovarian cyst containing ectodermal, endodermal, and mesodermal elements (i.e., skin, hair, teeth, and neural tissue)?

Teratoma (dermoid cyst)

What syndrome in children involves hypofunction of the adrenal glands due to bilateral hemorrhagic infarctions, which are most commonly associated with meningococcemia?

Waterhouse-Friderichsen syndrome

Ring-enhancing lesions seen on computed tomography (CT) scan of the brain in an HIV-positive individual are associated with what infection?

Toxoplasmosis (although you need to rule out cerebral abscess due to other organisms)

What extracellular protein forms subunits called ß-pleated sheets and when stained with Congo Red has an apple green birefringence under polarized light?

Amyloid

What renal calculus is associated with ureasplitting bacteria?

Magnesium ammonium phosphate (struvite)

With what two patiiologies is a honeycomb lung associated?

1. Asbestosis 2. Silicosis

234 USMLE Step I Recall

What syndrome has multiple adenomatous colonic polyps and CNS ghomas?

Turcot syndrome

Which form of melanoma carries the worst prognosis?

Nodular melanoma

What testicular tumor of infancy is characterized by elevated a-fetoprotein levels and "SchUler-Duval bodies"?

Yolk sac tumor

What is the pentad of thrombotic thrombocytopenic purpura (TTP)?

1. 2. 3. 4. 5.

What malabsorption syndrome presents with abdominal distention, bloating, flatulence, diarrhea, steatorrhea, and weight loss shortly after eating bread products?

Celiac sprue (gluten-sensitive enteropathy)

What AR disease involves a decreased amount of sphingomyelinase, massive organomegaly, zebra bodies, and foamy histiocytes microscopically and is associated with chromosome l i p ?

Niemann-Pick disease

What is a circumscribed, flat, nonpalpable pigmented change up to 1 cm?

Macule (e.g., a freckle)

ANAs and anti-SCL-70 antibodies are diagnostic of what disease?

Scleroderma

The onset of burning epigastric pain immediately

Gastric ulcer

Neurologic symptoms Renal failure Thrombocytopenia Fever Microangiopathic hemolytic anemia

Pathology 235

after eating describes what type of PUD? What vasculitis presents in a 30-year-old Asian female having visual field deficits, dizziness, decreased blood pressure, and weakened pulses in the upper extremities?

Tikayasu's arteritis (medium- to large-sized vessels)

Spider angiomas, palmar erythema, gynecomastia, testicular atrophy, encephalopathy, abnormalities in clotting factors, and portal HTN are all associated with what disorder?

Cirrhosis

In what rare AR disorder do you see neutropenia, defective degranulation, and delayed microbial killing due to a problem in Chemotaxis and migration?

Chédiak-Higashi syndr

What chromosome is mutant in patients with cystic fibrosis?

Patients with cystic fibrosis have a mutation in the chloride channel protein in the CFTR gene on chromosome 7.

HTN in the upper extremities, hypotension in the lower extremities, and a radial-femoral delay are associated with what vascular pathology?

Postductal coarctation of the aorta (adult)

Hereditary angioneurotic edema (AD) presents with local edema in organs (e.g., gastrointestinal [GI], skin, respiratoiy tract). What enzyme deficiency causes increased capillary

CI esterase inhibitor (C1INH)

236 USMLE Step I Recall

permeability due to a release of vasoactive peptides? True or false—hve vaccines are contraindicated in patients with SCID.

Tme

What pathology is associated with "lytic punched-out lesions," pathologic fractures, nerve compression, Bence-Jones proteinuria, hypercalcemia, renal failure, and amyloidosis?

Multiple myeloma

46XX karyotype and female internal organs with virilized external genitalia result in what condition?

Female pseudohermaphrodite

What is your diagnosis of a young, asymptomatic, thin female with a midsystolic click on cardiac auscultation?

Mitral valve prolapse

What tumor spreads via the lymphatic system into the peritoneum, rectal shelf, and both ovaries, secondary to metastatic gastric carcinoma?

Kmkenberg's tumor

Name the AD disease associated with chromosome 15 in which the patient has long extremities, lax joints, pigeon chest, and posterior mitral leaflet prolapse, and is p r o n e to developing dissecting aortic aneurysm.

Marfans syndrome

What form of hemophilia is X-linked recessive and is due to a deficiency in factor IX?

Hemophilia B

Pathology 237

In what disease do you see horseshoe kidneys, rockerbottom feet, low-set ears, micrognathia, and mental retardation?

Edwards syndrome (trisomy IS)

A 70-year-old man presents with the complaint of urinaiy urgency, nocturia, hesitancy, postvoid dribbling, urinaiy retention, and a PSA result of 6.5 ng/mL. What is your diagnosis?

BPH; although an argument can be made for prostatic cancer (you should expect a much higher PSA result), these are "buzzwords" for BPH. Prostatic cancer is usually silent until late in the disease when obstructive symptoms begin to occur.

In what type of GN, seen in the 2- to 6-year-old age group, is there albuminuria and effacement of the visceral epithelial foot process with no deposits?

Minimal change disease (lipoid nephrosis)

What is the most common location of a duodenal ulcer? Of a gastric ulcer?

Duodenal: anterior wall of the first part of the duodenum; gastric: lesser curvature of the antrum ol the stomach

How many grams of protein need to be excreted in 24 hours for the diagnosis of nephrotic syndrome to be made?

s.3.5 g/day of protein, along with generalized edema, hvpoalbuminemia, and hvperlipidemia

What thyroiditis presents as a tender, enlarged, firm thyroid gland, usuallypreceded by an upper respiratory viral illness?

de Queivain's thyroiditis

Name the six vitamin K-dependent coagulation factors.

Factors II, VII, IX, and X, and protein C and S.

What CNS tumor commonly presents with tinnitus and hearing loss?

Schwannoma

238 USMLE Step I Recall

How many café-au-lait spots need to be present prior to the diagnosis of neurofibromatosis type 1?

At least six

What are the three leftto-right shunts?

1. VSD 2. Atrial septal defect (ASD) 3. Patent ductus arteriosus (PDA)

Do the following structures pick up stain from hematoxylin or eosin: Nuclei?

Hematoxylin

Nucleoli?

Hematoxylin

Cytoplasm?

Eosin

Collagen?

Eosin

Red blood cells (RBCs)?

Eosin

Calcium?

Hematoxylin

Bacteria?

Hematoxylin

Fibrin?

Eosin

Thyroid colloid?

Eosin

An 80-year-old woman presents to you with rightsided temporal headache, facial pain and blurred vision on the affected side, and an elevated erythrocyte sedimentation rate (ESR). What is your diagnosis?

Temporal arteritis (giant cell arteritis)

What syndrome has loss of deep tendon reflexes, muscle weakness, and ascending paralysis preceded by a viral illness?

Guillain-Barré syndrome

Pathology 239

What three criteria allow you to differentiate an ulcer from an erosion or carcinoma?

Name the nephritic disease based on the immunofluorescent staining below: Mesangial deposits of IgA andC3

< 3 cm 2 Clean base Level with the surrounding mucosa

IgA nephropathy (Berger's disease)

Smooth and hnear pattern of IgG and C3 in the GBM

Goodpasture's disease

Granular deposits of IgG, IgM, and C3 throughout the glomerulus

Postinfectious GN

What autoimmune liver disease is characterized by being a middle-aged female who presents with jaundice, pruritus, fatigue, xanthomas, increased direct bilirubin levels, and antimitochondrial Abs?

Primary biliary cirrhosis

Which is more commonly associated with primary sclerosing cholangitis: ulcerative colitis or Crohn's disease?

Ulcerative colitis

Name the four right-to-left congenital cardiac shunts?

Truncus arteriosus, Tricuspid atresi Transposition of the great vessels, a Tetralogy of Fallot. (They all begin with "T".)

Mediterranean type of what deficiency is a result of eating fava beans?

G-6-PD deficiency

240 USMLE Step I Recall

What childhood pathology involves anterior boxving of the tibia, epiphyseal enlargements, and costochondral widening with the endochondral bones being affected?

Rickets

What is the tenn for an abnormal amount of collagen type III that produces a large bulging scar, seen primarily in blacks?

Keloid

What leukemia is the most indolent of all leukemias, affects persons older than 55 years of age, and is associated with trisomy 12? (Hint: 95% are B-cell neoplasms.)

Chronic lymphocytic leukemia (CLL)

Bitter almond-scented breaths are associated with what fonn of poisoning?

Cyanide

What artery is the most common site of infarctions in the cerebral circulation?

Middle cerebral artery

Bdateral involvement of the uvea, parotid, submandibular, and submaxillary gland in a patient with sarcoidosis is tenned what syndrome?

Mikulicz syndrome

With which pituitary adenoma is an elevated somatomedin C level associated?

Growth hormone (GH)-producing adenoma

IgG Abs against Bh antigens, positive direct Coombs' test, and splenomegaly are seen with what fonn of anemia?

Autoimmune hemolytic anemia

Pathology 241

What is the term for a benign, melanocytic tumor associated with sun exposure that presents as tan to brown colored and has sharply defined wellcircumscribed borders?

Benign nevi (mole)

A mother brings her 2-weekold infant to the ER with the symptoms of regurgitation and vomiting after eating, peristaltic waves visible on the abdomen, and a palpable mass in the right upper quadrant; what would you diagnose?

Pyloric stenosis

A 30-year-old woman presents to your office with bilateral, multiple breast nodules that vary with menstruation and have cyclical pain and engorgement. What is your diagnosis?

Fibrocystic change of the breast. This highlights the distinguishing features from breast cancer, which are commonly unilateral, single nodule; no variation with menstruation; and no changes during pregnancy.

What is the "Rules of 2* for Meckel's diverticulum?

2% of population, 2 cm long, 2 feet from ileocecal valve, 2 years old, and 2% of carcinoid tumors

Cytoplasmic remnants of RNA in RBCs that are seen in lead poisoning is known as?

Basophilic stippling

What disease is X-linked recessive, presents with eczema thrombocytopenia and an increased chance of developing recurrent infections, involves a decrease in serum IgM and the T cell-dependent paracortical areas of the

Wiskott-Aldrich syndrome

242 USMLE Step I Recall lymph nodes, and means that the patient is likely to develop malignant lymphoma? What is it called when the cervical lymph node is involved in TB?

Scrofula

What pathology has bilateral periventricular plaques, perivascular inflammation, and demyelination and is thought to be autoimmune or brought on by a viral infection?

Multiple sclerosis

What virus is associated with both nasopharyngeal carcinoma and Burkitt's lymphoma?

EBV

English chimney sweeps have an increased risk of developing what cancer?

Scrotal cancer, due to the increased exposure to polycyciic aromatic hydrocarbons

What is a palpable, elevated solid mass up to 0.5 cm?

Papule

What type of Ehlers-Danlos syndrome is X-hnked recessive and is caused by a defect in copper metabolism, causing a problem with crosslinking collagen and elastin fibers?

Type IX

What type of Ehlers-Danlos syndrome involves decreased activity of lysyl hydroxylase and affects collagen types I and III the most?

Type VI (AR)

Pathology 243

What small- to metZtum-sized vasculitis is seen in a 35-year-old male who is a heavy smoker presenting with claudication symptoms in the upper and lower extremities?

Buerger's disease (thromboangiitis obliterans)

What is the triad of Reiter's syndrome?

1. Peripheral arthritis 2. Conjunctivitis 3. Nongonococcal urethritis

What bilateral, AR disorder, seen in infancy as progressive renal fadure, has multiple small cysts at right angles to the cortical surface?

Polycystic kidney disease of childhood

What urinary metabolite is increased in patients with carcinoid syndrome?

5-hydroxyindoleacetic acid (5-HIAA)

What type of Ehlers-Danlos syndrome involves a deficiency in procollagenN-peptidase?

Type VII (AR)

Chronic necrotizing pulmonary infections resulting in permanent airway dilation, associated with Kartagener's syndrome is known as?

Bronchiectasis

What is the triad of Felty's syndrome?

Neutropenia, splenomegaly, and rheumatoid arthritis

What syndrome is due to antiglomerular basement membrane (anti-GBM) Abs directed against the lung and kidneys?

Goodpasture's syndrome

244 USMLE Step I Recall

What bronchogenic carcinoma is commonly associated with Addison's disease?

Squamous cell carcinoma

Pus in the pleural space is known as?

Empyema

What two components of the complement cascade are anaphylatoxins?

1. C3a 2. C5a (causes mast cells to secrete histamine)

What type of crystals are associated with gout?

Monosodium urate crystals

Squamous to columnar metaplasia of the distal esophagus secondary to chronic inflammation is known as?

Barrett's esophagus has an increased risk of developing adenocarcinoma of die esophagus.

What seronegative spondyloarthropathy is seen in HLA-B27-positive young females and presents with the triad of conjunctivitis, urethritis, and arthritis, affecting the knees and ankles?

Reiter's syndrome

What pancreatic islet cell tumor is associated with multiple endocrine neoplasia (MEN) I syndrome?

Gastrinomas

What are the four signs of acute inflammation?

Rubor (red), dolor (pain), calor (heat), and tumor (swelling), and sometimes there is even loss of function

What seronegative spondyloarthropathy is seen in HLA-B27-positive young men, involves the sacroiliac

Ankylosing spondylitis

Pathology 245

joints, has no subcutaneous nodules, and has a "bamboo" spine appearance on radiography? What liver tumor is associated with OCP use? Name the cancer associated with the following chemical agents (some may have more than one answer): Alkylating agents Aromatic amines and azo dyes

Hepatic adenomas

Leukemias and lymphomas Hepatocellular carcinoma

Arsenic

Squamous cell carcinoma (skin, lung) and angiosarcoma of the liver

Asbestos

Mesothelioma and broiiehogeiiic carcinoma

Naphthylamine Benzenes Vinyl chloride Chromium and nickel

Bladder cancer Leukemias Angiosarcoma of the liver Bronchogenic carcinoma

Polycyclic aromatic hydrocarbons

Bronchogenic carcinoma

Nitrosamines

Gastric cancer

What B-cell neoplasm is seen in males with massive splenomegaly, diy tap on bone marrow aspirations, and stains positive for tartrateresistant acid phosphatase (TBAP)?

Hairy cell leukemia

246 USMLE Step I Recall

What form of coarctation of the aorta is associated with Turner's syndrome?

Preductal (infantile)

Neurologic signs consistent with a cerebrovascular accident, but lasting 24 hours with full recovery, is known as?

Transient ischemic attack (TIA)

What benign, solitary papillary growth within the lactiferous ducts of the breast commonly presents with bloody nipple discharge?

Intraductal papilloma

A marfanoid patient presents with tearing retrosternal chest pain radiating to her back. What is your first diagnosis?

Dissecting aortic aneurysm (Myocardial infarction [MI] is also high on the list, but the italicized words are "buzzwords" to look for dissection.)

What systemic, chronic inflammatory disease is commonly seen in females aged 20-50 and presents as a progressive, symmetric arthritis affecting the hands, wrists, knees, and ankles that improves with increased activity?

Rheumatoid arthritis

What pneumoconiosis is associated with exposure to the following occupations or materials: Miners, metal grinders, and sandblasters?

Silicosis

Aerospace industry, nuclear reactors?

Berylliosis

Shipyards, brake linings, insulation, and old building construction?

Asbestosis Note: Coal worker's pneumoconiosis is synonymous with black lung disease, an upper lobe occupational disorder.

Pathology 247

Seeing negatively biréfringent, needle-shaped crystals from a joint aspiration of the great toe, you diagnose what fonn of arthritis?

Gout

Are the following major or minor Jones criteria of rheumatic fever: Fever?

Minor

Migratory polyarthritis?

Major

Subcutaneous nodules?

Major

Elevated acute-phase reactants (i.e., ESR)?

Minor

Arthralgias?

Minor

Pericarditis?

Major

Erythema marginatum?

Major

Sydenham's chorea?

Major

IgE-mediated mast cell release, C3a and C5a, and interleukin (IL)-l all trigger the release of what vasoactive amine?

Histamine

What pulmonary pathology, most commonly associated with smoking, results in enlarged, overinflated lungs owing to the destruction of the alveolar walls with diminished elastic recoil?

Emphysema

Goodpasture's Ag is a component of what type of collagen?

Type IV collagen

248 USMLE Step I Recall

What disease involves microcephaly, mental retardation, cleft lip or palate, and dextrocardia?

Patau's syndrome (trisomy 13)

What joint is affected causing Heberden's nodes in osteoarthritis?

Distal inteiphalangeal (DIP) joint (Bouchard's nodes are in die proximal inteiphalangeal [PIP] joint)

Gastrin-producing islet cell tumor resulting in multiple, intractable peptic ulcers is seen in what syndrome?

Zollinger-Ellison syndrome

With what form of cancer is diethylstilbestro] (DES) associated?

Vaginal clear cell adenoma

What inflammatory bowel disorder is continuous with extensive ulcerations and pseudopolyps and is associated with HLA-B27?

Ulcerative colitis

What components of the complement cascade form the membrane attack complex (MAC)?

C5b-C9

Patients with sickle cell anemia are at increased risk of infection from what type of organisms?

Encapsulated barter

What is die "Rules of 10" regarding pheochromocytoma?

10% are bilateral, 10% malignant, and 10% familial, in children, and occur outside the adrenal gland

"Blue sclera" is seen in what hereditary bone disorder?

Osteogenesis imperfecta

When a venous embolus enters into the arterial system, it is termed what?

Paradoxical emboli are most commonly through a patent septal defect in the heart.

Pathology 249 What autosomal recessive (AR) syndrome is due to a deficiency of glycoprotein lb, resulting in a defect in platelet adhesion?

Bernard-Soulier syndrome

What protein-losing enteropadiy has grossly enlarged rugal folds in the body and fundus of the stomach in middleaged males, resulting in decreased acid production and an increased risk of gastric cancer?

Ménétrier's disease

What skin carcinoma is a superficial dermal infiltrate of T lymphocytes seen in males older than 40 years of age and presents as scaly, red patches or plaques?

Mycosis fungoides (cutaneous T-cell lymphoma)

How many segments in a neutrophilic nucleus need to be present for it to be referred to as hypersegmented?

Five or more lobes

What are the five reasons for normochromic normocytic anemia with a normal MCV and an elevated reticulocyte count?

1. 2. 3. 4. 5.

What is the triad of Meig's syndrome?

1. Right-sided hydrothorax 2. Ascites 3. Ovarian fibroma

What malignant tumor of die skin is associated with Birbeck granules?

Histiocytosis X

Finding well-demarcated erythematous plaques with silvery scales on the knees,

Psoriasis

Autoimmune hypersplenism Trauma Anemia Spherocytosis Sickle cell anemia

250 USMLE Step I Recall

elbows, and scalp as well as nail bed pitting and discoloration leads to the diagnosis of what? What variant of Hodgkin's lymphoma can be localized or widespread and has an intermediate prognosis with lymphocytes, eosinophils, plasma cells, histiocytes, and Reed-Stemberg cells?

Mixed cellularity

What disease involves an adenosine deaminase deficiency, B- and T-cell deficiency, and defective IL-2 receptors?

Severe combined immunodeficiency (SCID)

What skin condition has irregular blotchy patches of hyperpigmentation on the face commonly associated with OCP use and pregnancy?

Melasma

Watery diarrhea, hypokalemia, and achlorhydria are associated with what pancreatic islet cell tumor?

VIPoma

Severe and protracted vomiting, resulting in linear lacerations at the gastroesophageal junction, is known as?

Malloiy-Weiss syndrome

What virus is associated with body cavity large B-cell lymphomas?

Human heipesvims 8 (HHV-8)

In what disease do you see atrophy of the frontal and temporal lobes ("walnut brain")?

Pick's disease

Pathology 251 What type of vasculitis affects children, has conjunctival involvement associated with skin rash and lymphadenopathy, and involves 70% of persons with coronary artery aneurysms?

Kawasaki disease Think kids, koronaiy, konjunctiva— Kawasaki.

D o w n w a r d displacement of t h e c e r e b e l l a r vermis and m e d u l l a c o m p r e s s i n g the fourth ventricle and leading to obstructive hydrocephalus is s e e n with what CNS d e v e l o p m e n t a l abnormality?

Arnold-Chiari malformation (type 2)

N a m e two of three e n z y m e s that protect t h e cell from oxygen-derived free radicals.

Superoxide dismutase, glutathione peroxidase, catalase

Cough with copious sputum production n e e d s to occur for h o w many months in h o w many years for the diagnosis of chronic bronchitis to b e m a d e ?

3 m o n t h s of symptoms in 2 c o n s e c u t i v e years

True or false—HPV infection increases the risk of d e v e l o p i n g squamous cell carcinoma of t h e penis.

T m e ; HPV serotypes 16 and 18 are risk factors for squamous cell carcinoma.

What rare vasculitis has the following characteristics: affects males a g e d 4 0 - 6 0 ; affects small arteries and veins; involves the nose, sinuses, lungs, and kidneys; and has cytoplasmic antineutrophilic cytoplasmic antibody (C-ANCA) and autoantibodies against proteinase 3?

Wegener's granulomatosis

252 USMLE Step I Recall

What enzyme is deficient in alkaptonuria?

Homogentisic oxidase

In what quadrant is breast cancer most commonly found?

Upper outer quadrant (left > right too!)

Name the hypochromic microcytic anemia based on the following lab values. Increased iron, decreased total iron binding capacity (TIBC), increased percent saturation, increased ferritin

Sideroblastic anemia

Decreased iron, TIBC, and percent saturation; increased ferritin

Anemia of chronic disease

Decreased iron, percent saturation, and ferritin; increased TIBC

Iron deficiency anemia

Nonnal iron, TIBC, percent saturation, and ferritin

Thalassemia minor

Podagra, tophi in the ear, and polymorphonucleocytes (PMNs) with monosodium u r a t e crystals are associated with what pathology?

Gout

What prion-associated CNS pathology presents as a rapidly progressive dementia with myoclonus, involuntary movements, and deadi within 6-12 months?

Creutzfeldt-Jakob disease

What amino acid (AA) is substituted for glutamic acid at position 6 on the ß chain

Valine

Pathology 253

in patients with sickle cell

What disease has the following characteristics: has autoantibodies to IgG; occurs in women more than men; and includes exophthalmos, pretibial myxedema, nervousness, heart palpitations, and fatigue?

Graves' disease

Can an acute MI be diagnosed by only looking at an ECG?

No; remember, tests do not diagnose; they confirm or refute your diagnosis. Also, you need two of three criteria to be met for an MI to be diagnosed: chest pain consistent and characteristic of an MI, elevated cardiac enzymes consistent with an MI, and ST segment elevation of 2 mm or more in at least two contiguous leads.

Name which hepatitis B serology markers— HB Ab IgG, HB Ab IgM, HBcAg, HBAb IgG, HB Ag, or HBV-DNA—are associated with the following "periods" Window period

HBcAb IgM

Immunization

HB Ab IgG

Prior infection

HBcAb IgG and HBsAb IgG

Acute infection

HBAb IgM, HBV-DNA, HBeAg, andHB s Ag

Chronic infection

HBcAb IgG, HBV-DNA, HBeAg, and HBsAg

What AR CNS disorder presents early in childhood with gait ataxia, loss of deep tendon reflexes, impaired

Friedreich's ataxia

254 USMLE Step I Recall vibratory sensation, hand clumsiness, and loss of position sense? What benign bone tumor is associated with Gardner's syndrome?

Osteoma

What transmural inflammatory bowel disease can be found from the mouth to anus; has noncaseating granulomas; is discontinuous (skip lesions); has a cobblestone appearance, Uiickening of the bowel wall, and linear fissures; and has aphthous ulcers with normal mucosa between?

Crohn's disease

The speckled appearance of the his in patients with Down syndrome is called what?

Bmshfield spots

Based on the following information, is the renal transplantation rejection acute, chronic, or hyperacute: Months to years after transplantation; gradual onset of hypertension (HTN), oliguria, and azotemia; seen as intimai fibrosis of the blood vessels and interstitial lymphocytes?

Chronic rejection

Immediately after transplantation; seen as a neutrophdic vascuhtis with thrombosis?

Hyperacute rejection

Weeks to months after transplantation; abrupt

Acute rejection

Pathology 255

onset of oliguria and azotemia; seen as neutrophilic vasculitis and interstitial lymphocytes? What pathology has the following signs: severe headache, palpitations with or without tachycardia, diaphoresis, anxiety, nervousness, and hypertensive episodes?

Pheochromocytoma

What pathology has the following signs: elevated levels of Ca 2 + , cardiac arrhythmias, bone resorption, kidney stones, and metastatic calcifications?

Primary hyperparathyroidism

A congenital absence of the ganghonic cells of the Auerbach's and Meissners plexuses in the rectum and sigmoid colon is known as?

Hirschsprung's disease (aganglionic megacolon)

When a large VSD leads to pulmonary HTN, right ventricular hypertrophy (RVH), and cyanosis due to right-to-left reversal of the shunt, it is known as?

Eisenmenger's syndrome (It can also occur with any Ieft-to-right shunt.)

Name the product(s) from arachidonic acid based on the following description: Vasodilation and inhibition of platelet aggregation, produced by vascular endothelium

Prostaglandin I, (PGI,)

Vasodilation

PGE 2 , PGD 2 , and PGF,

Pain and fever production

PGE,

256 USMLE Step I Recall

Vasoconstriction and platelet aggregation, produced by platelets

Thromboxane A, (TXA,)

Chemotaxic for neutrophils

Leukotriene B4 (LTB4)

Vasodilation, bronchospasm, and increased vascular permeability

LTC4, LTD4, and LTE4

What glycogen-storage disease is due to the following enzyme deficiencies: Lysosomal glucosidase (acid maltase)?

Pompes disease

Muscle phosphorylase?

McArdle's syndrome

Glucose-6-phosphatase?

von Gierke's disease

What is the syndrome associated with women of short stature with a web neck, a low posterior hairline, streaky ovaries, and preductal coarctation of the aorta?

Turner's syndrome

What is the first sign of megaloblastic anemia on a peripheral blood smear?

Hypersegmented neutrophils

What rare disorder presents as a large, hard, irregular thyroid gland due to fibrous proliferation of connective tissue in the thyroid gland and extends to adjacent structures?

Riedel's thyroiditis

What disease is found in persons in the 20- to 40-year-old age range; affects women more than

Giant cell tumor of the bone

Pathology 257 men; occurs at the epiphysis of the knee; and is seen as a "soap bubble" appearance on a radiography? What condition results in a "strawberry gallbladder"?

Cholesterolosis

What condition is manifested by büateral sarcoidosis of the parotid glands, submaxillary gland, and submandibular gland with posterior uveal tract involvement?

Mikulicz's syndrome

In what AR disease do you see a decrease in adenine triphosphatase (ATPase) activity of dynein arms in cilia, also known as immotile cilia syndrome?

Kartageners syndrome

What type of GN has C3 deposits in irregular granular/linear foci and intramembranous deposits of unknown material?

MPGN type II

What autoimmune syndrome is characterized by keratoconjunctivitis, corneal ulcers, xerostomia, and an increased risk of high grade B-cell lymphomas? What two Ab tests are used in making the diagnosis?

Sjogren's syndrome; SS-A (Ro) and SS-B (La)

After he had been traveling in a plane across the Atlantic Ocean, an obese male presents to the emergency room (ER) with right leg swelling and sudden onset of shortness of breath; you immediately diagnose?

Pulmonaiy embolism due to a deep vein thrombosis (DVT) (This is not an absolute, but a "classic" description.)

258 USMLE Step I Recall

Name the type oí exudate, given the following examples: Sunburn

Serous exudates

Uremic pericarditis

Fibrinous exudates

Parasitic infection

Eosinophilic exudates

Diphtheria infection

Pseudomembranous exudates

Meningococcal infection

Purulent exudates

Rickettsial infection

Hemorrhagic exudates

What is the name for the following RBC indices: The coefficient of variation of the RBC volume?

Bed cell distribution width (RDW)

Average mass of the Hgb molecule/RBC?

Mean coipuscular Hgb (MCH)

Average volume of a RBC?

MCV

Average Hgb concentration/given volume of packed RBCs?

Mean coipuscular Hgb concentration (MCHC)

What tumor marker is associated with seminomas?

Placental alkaline phosphatase

What is the tumor at the bifurcation of the right and left hepatic ducts known as?

Klatskin's tumor

"Starry sky" appearance of macrophages is pathognomonic of what lymphoma?

Burkitt's lymphoma

What gene inhibits apoptosis by preventing the release of cytochrome c from mitochondria?

Bcl-2

Pathology 259

What renal pathology is associated with an alteration of the basement membrane and mesangial cell proliferation, along with a tram-track appearance under the microscope?

MPGN

What are the five components of portal HTN?

1. 2. 3. 4. 5.

What CNS tumor arises from Rathke's pouch?

Craniopharyngioma

What fonn of necrosis is caused by the actions of lipases on adipose tissue, seen in acute hemorrhagic pancreatitis?

Fat necrosis

What leukemia is seen in the 25- to 60-year-old age group and is associated with chromosome 9,22 translocation, bcr-abl oncogene, and blast crisis?

Chronic myelocytic leukemia (CML)

What GN is highly associated with hepatitis B and C infections?

Membranoproliferative glomerulonephritis (MPGN)

What is a sudden loss of oxygen to tissue, causing death of the cells because of a cessation of blood flowing to a particular area, known as?

Coagulative necrosis (most common in heart and kidneys)

What syndrome has elevated follicle-stimulating hormone (FSH) and leuteinizing honnone (LH) levels with

Klinefelter's syndrome

Caput medusae Esophageal varices Ascites Splenomegaly Hemorrhoids

260 USMLE Step I Recall

decreased testosterone levels and 47XXY karyotype? In what AD pathology is there a derangement of the epiphyseal cartilage growth, causing a large skull and a normal-sized vertebral column?

Achondroplasia

What is the term for thickened, hyperpigmented sldn in the axillae, groin, and skin folds associated with mahgnancies, obesity, and DM?

Acanthosis nigricans

What condition results in the following CSF results: Opening pressure 70-180 mm H 2 0 ; 0-10 WBCs (monocytes); glucose 45-85, protein 15-45?

Normal values

Opening pressure 450 mm H 2 0 ; five WBCs (90% lymphocytes); normal glucose and protein levels?

Brain abscess

Opening pressure 100 mm H 2 0 ; 120 WBCs (90% lymphocytes); normal glucose levels; protein 17?

Viral meningitis

Opening pressure 250 mm H 2 0 ; 250 WBCs (90% lymphocytes); glucose 35; protein 100?

TB meningitis

Opening pressure 400 mm H , 0 ; 8500 WBCs

Bacterial meningitis

Pathology 261

(90% PMNs); glucose 15; protein 120? What aspect of the complement cascade is defective if a patient constantly presents with recurrent infections with Neisseria gonorrhoeae or Neisseria meningitidis?

MAC (C5-C9)

What AD GI neoplasia presents with multiple adenomatous polyps, osteomas, fibromas, and epidural inclusion cysts?

Gardner's syndrome

Trae or false—patients with Turner's syndrome have no Barr bodies.

Tme; remember, the second X chromosome is inactivated and is the Barr body. Turner's syndrome has only one X chromosome.

What viral infection in patients with sickle cell anemia results in aplastic crisis?

Parvovirus B19

What parasitic infection is associated with cholangiocarcinoma?

Clonorchis sinensis

What parasite is associated with squamous cell carcinoma of the urinaiy bladder?

Schistosoma haematobium

Name the hepatitis virus based on the following information: Small circular RNA virus with defective envelope Enveloped RNA fiavi virus

Hepatitis D

Hepatitis C

262 USMLE Step I Recall

Naked capsid RNA caliciviras

Hepatitis E

Enveloped DNA hepadnavirus

Hepatitis B

Naked capsid RNA picorn virus

Hepatitis A

What are the following risk factors characteristic of: late menopause, early menarche, obesity, nulliparity, excessive estrogen, genetic factor p53, and brc-abl?

Breast cancer

What coronary artery vasculitis is seen in Japanese infants and children younger than 4 years old presenting with an acute febrile illness, conjunctivitis, maculopapular rash, and lymphadenopathy?

Kawasaki's disease

Inflamed, thickened skin on the breast with dimpling associated with cancer is known as?

Peau d'orange

What leukemia is seen in the 15- to 39-year-old age group, has blasts with myeloperoxidase-positive granules, tends to invade tissues, and is associated with a poor prognosis?

Acute myelocytic leukemia (AML)

Chapter 8

Power Review

BUZZWORDS Calf pseudohypertrophy

Duchenne muscular dystrophy

Kayser-Fleischer rings

Wilson disease

Aschoff bodies

Rheumatic fever

Curschmann spirals

Bronchial asthma (whorled mucous plugs)

Charcot-Leyden crystals

Bronchial asthma (eosinophil membranes)

Keratin pearls

Squamous cell carcinoma

Bence-Jones proteinuria

Multiple myeloma

Russell bodies

Multiple myeloma

White blood cells (WBCs) in the urine

Acute cystitis

Red blood cells (RBCs) in the urine

Bladder carcinoma

RBC casts in the urine

Acute glomerulonephritis

WBC casts in the urine

Acute pyelonephritis

Renal epithelial casts in the urine

Acute toxic or viral nephrosis

Waxy casts

Chronic end-stage renal disease

Signet ring cells

Gastric carcinoma

Heinz bodies

G-6-PD deficiency

Mallory bodies

Chronic alcoholic 263

264 USMLE Step I Recall Auer rods

Acute myelocytic leukemia-M3 (AML-M3)

Stariy sky pattern

Burkitt lymphoma

Birbeck granules

Histiocytosis X

R e e d - S t e r n b e r g cells

Hodgkin lymphoma

Call-Exner b o d i e s

Granulosa/thecal cell tumor of the ovary

C o w d i y type A b o d i e s

Heipesvims

O i p h a n Annie ceUs

Papillary carcinoma of the ovary

Streaky ovaries

Turner syndrome

B l u e - d o m e d cysts

Fibrocystic change of the breast

R e i n k e crystals

Leydig cell tumor

Schiller-Duval b o d i e s

Yolk sac tumor

C o d m a n triangle o n radiograph

Osteosarcoma

Councilman b o d i e s

Toxic or viral hepatitis

B l u e sclera

Osteogenesis imperfecta

Soap b u b b l e a p p e a r a n c e o n radiograph

Giant cell tumor of the bone

Pseudorosettes

Evving sarcoma

Neurofibrillary tangles

Alzheimer disease

Homer-Wright rosettes

Neuroblastoma

Lewy bodies

Parkinson disease

Lucid interval

Epidural hematoma

Bloody tap o n lumbar puncture

Subarachnoid hemorrhage

Power Review 265

Pseudopalisades

Glioblastoma multiforme

Senile plaques

Alzheimer disease

Café-au-lait spot on the skin

Neurofibromatosis

CHROMOSOMES

Name the associated chromosomal translocation: Chronic myelocytic leukemia (CML)

Chromosome 9,22 (Philadelphia chromosome)

Ewing sarcoma

Chromosome 11,22

Adult familial polyposis

Chromosome 5,21

Burkitt lymphoma

Chromosome 8,14

Acute promyelocytic leukemia (M3)

Chromosome 15,17

Follicular lymphoma

Chromosome 14,18

Name the associated chromosome: Cri-du-chat

Chromosome 5p

Patau syndrome

Chromosome 13

Huntington disease

Chromosome 4p

Familial hypercholesterolemia

Chromosome 19

Gaucher disease

Chromosome 1

Niemann-Pick disease

Chromosome l i p

Tay-Sachs disease

Chromosome 15q

Cystic fibrosis

Chromosome 7

Albinism

Chromosome l i p

266 USMLE Step I Recall

Chronic lymphocytic leukemia (CLL)

Chromosome 12

Marfan disease

Chromosome 15

Neurofibromatosis type 1

Chromosome 17

Neurofibromatosis type 2

Chromosome 22q

Down syndrome

Chromosome 21

Edward syndrome

Chromosome 18

von Hippel-Lindau disease

Chromosome 3p

What is the most common chromosomal disorder?

Down syndrome

What is the most common cause (MCC) of inherited mental retardation?

Down syndrome

What is the most common lysosomal storage disorder?

Gaucher disease

What chromosome is mutant in patients with cystic fibrosis?

Patients with cystic fibrosis have a mutation in the chloride channel protein in the CFTR gene on chromosome 7.

Familial hypercholesteremia is due to a mutation in the low-density lipoprotein (LDL) receptor gene. What chromosome carries it?

Chromosome 19

What chromosome is associated with the autosomal recessive (AR) form of severe combined immunodeficiency (SCID)?

Chromosome 20q (deficiency in adenosine deaminase)

Power Review 267

On what chromosome is the adenomatous polyposis coli gene?

Chromosome 5q21

VITAMINSAND MINERALS What mineral is associated with impaired glucose tolerance?

Chromium (Cr)

What mineral is associated with hypothyroidism?

Iodine (I)

What mineral is an important component of the enzyme xanthine oxidase?

Molybdenum (Mb)

What vitamin deficiencyproduces angular stomatitis, glossitis, and cheilosis?

Riboflavin (B,) deficiency

What vitamin is a component of the coenzyme thiamine pyrophosphate (TPP)?

Thymine (B,)

Avidin decreases the absorption of what vitamin?

Biotin. Avidin is found in raw egg whites.

What are the four Ds of niacin deficiency?

1. Diarrhea 2. Dermatitis 3. Dementia

4. Death What mineral is an important component of glutathione peroxidase?

Selenium (Se)

What mineral deficiency in children is associated with poor growth and impaired sexual development?

Zinc (Zn) deficiency

268 USMLE Step I Recall

What mineral, via excessive depositions in the hver, causes hemochromatosis?

Iron (Fe)

What vitamin is needed for the production of heme?

Pyridoxine (B6

What vitamin is a component of the enzymes fatty acid synthase and acyl CoA?

Pantothenic acid

What vitamin deficiency produces homocystinuria and methylmalonic aciduria?

Cyanocobalamin (B15) deficiency (Folic acid deficiency has only homocystinuria as a sign.)

What vitamin deficiency is evidenced by poor wound healing, loose teeth, bleeding gums, petechiae, and ecchymosis?

Ascorbic acid (vitamin C) deficiency (These are the signs of scurvy.)

What vitamin is given as prophylactic treatment for patients with alcoholism?

Thiamine (Bj), to prevent Wernicke encephalopathy and Korsakoff encephalopathy

What are the three carboxylase enzymes that require biotin?

Pyruvate, acetyl CoA, and propionyl CoA carboxylase

What vitamin requires intrinsic factor (IF) for absorption?

Cyanocobalamin (B ß )

What mineral is a component of cytochrome a/a3?

Copper

Leukopenia, neutropenia, and mental deterioration are signs of what mineral deficiency?

Copper

What vitamin deficiency causes a glove-and-stocking

Pyridoxine (Bß) deficiency

Power Review 269

neuropathy seen in alcoholics? What mineral deficiency involves blood vessel fragility?

Copper deficiency

Megaloblastic anemia and thrombocytopenia are signs of what vitamin deficiency?

Folic acid deficiency

What vitamin deficiency can result in high-output cardiac failure?

Thiamine

ANTIDOTES Name the antidote: Oxygen Carbon monoxide (CO) Dimercaprol, penicillamine Mercury Pyridoxine Isoniazid Physostigmine Atropine Dimercaprol, D-penicillamine Arsenic Antidigoxin Fab fragments Digoxin Dimercaprol Gold Ethyl alcohol Ethylene glycol Naloxone, naltrexone Opioids Atropine, 2-PAM (pralidoxime) Organophosphates Warfarin Copper Heparin

Vitamin K D-Penicillamine Protamine sulfate

270 USMLE Step I Recall

Iron

Deferoxamine, penicillamine

Cyanide

Amyl nitrate, sodium nitrate, or sodium thiosulfate

Methyl alcohol

Ethyl alcohol

Acetaminophen

N-Acetylcysteine

Nitrates

Methylene blue

Lead

Penicillamine, EDTA (calcium disodiun edetate), or dimercaprol

Benzodiazepines

Flumazenil

Acetylcholinesterase (AChE) inhibitors

Atropine with pralidoxime

Anticholinergics

Physostigmine

A known IV drag abuser goes to the emergency room (ER) with respiratoiy depression and pinpoint pupds and in a semicomatose state. What overdose do you suspect, and what agent will you give to reverse its effects?

The triad of respiratoiy depression, pinpoint pupils, and coma in an IV drug abuser suggests opioid overdose (e.g., heroin), and you should give naloxone.

Cherry red intoxication is associated with what form of poisoning?

CO

What toxicities are caused by the following agents: Occupational nitrous

Anemia

oxide exposure? Nephrotoxicity Methoxyflurane? Hepatitis, with or without necrosis Halothane?

Power Review 271

What are the first signs of overdose from phénobarbital?

Nystagmus and ataxial

What are the three signs of morphine overdose?

Pinpoint pupils, decreased respiratoiy rate, and coma

What is the major pulmonary side effect of (i-activators?

Respiratory depression

What are the two side effects of opioids to which the user will not develop tolerance?

Constipation and miosis

MOST COMMON Name the most common cause: Blindness worldwide

Chlamydia trachomatis

Blindness in the United States

Diabetes mellitus

Dementia in persons aged 60-90 years

Alzheimer disease

Name the most common type: Learning disability

Dyslexia

Mental retardation

Fetal alcohol syndrome

Phobia

Stage fright (discrete performance anxiety)

Psychiatric disorder in women of all ages

Anxiety disorders; for men it is substance abuse.

Specific phobia Name the most common cause: Chronic pancreatitis

Public speaking (Remember: stage fright)

Alcohol abuse

272 USMLE Step I Recall

Infectious pancreatitis

Mumps

Insomnia

Depression

Hospitalization in children younger than 1 year of age

Respiratoiy syncytial vims (RSV)

Croup

Parainfluenza vims

A cold in the winter and summer

Coronavirus

A cold in the spring and fall

Rhinovirus

Which virus is the MCC of a cold in the summer and fall? Winter and spring? Name the most common type or cause: Restrictive cardiomyopathy Death in the United States

Rhinoviruses for summer and fall; coronavimses for winter and spring

Amyloidosis

Ischemic heart disease (myocardial infarctions [Mis])

Sudden cardiac death

Ventricular fibrillation (V fib)

Right heart failure

Left heart failure

Childhood heart disease in the United States

Congenital heart disease

Cyanotic heart disease

Tetralogy of Fallot

Subacute bacterial endocarditis

Streptococcus uiridans

Acute bacterial endocarditis

Staphylococcus aureus

Viral pneumonia leading to death

RSV

Power Review 273

Infection in a patient on a ventilator

Pseudomonas aeruginosa

Bronchiolitis in children

RSV

Pulmonary hypertension (HTN) in children

Ventricular septal defect (VSD)

Beversible HTN in the United States

Alcohol abuse

Spontaneous pneumothorax

Emphysematous bleb

Nonorganic pneumoconiosis

Asbestosis

Cellular injury

Hypoxia

Hypoxia Ischemia Lobar pneumonia Streptococcus pneumoniae Lung abscess Cirrhosis in the United States

Aspiration Alcohol consumption

Nephrotic syndrome

Membranoproliferative glomemlonephritis (MGN)

Nephrotic syndrome in children

Lipoid nephrosis

Liver transplantation in adults

Alcoholic cirrhosis

Urinary tract infections (UTIs)

Escherichia coli

Urinary tract obstruction

Benign prostatic hyperplasia (BPH)

Painless hematuria

Renal cell carcinoma

274 USMLE Step I Recall

Hematuria

Infection

Glomerulonephritis (GN) in the world

IgA nephropathy

Nephritic syndrome in children

Minimal change disease

Nephritic syndrome in adults

MGN

Acute renal failure in the United States

Acute tubular necrosis

Acute tubular necrosis

Ischemic

Diarrhea in children

Rotavirus

Intestinal obstructions in adults

Adhesions and hernias

Neonatal bowel obstruction

Hirschsprung disease

Rectal bleeding

Diverticulosis

Mallory-Weiss syndrome

Alcoholism

Anovulation

Polycystic ovaries

Breast lump in females

Fibrocystic change of the breast

Hematologic cause of papillary necrosis

Sickle cell disease

Panhypopituitarism

Sheehan syndrome

Cushing syndrome

Pituitaiy adenoma

Noniatrogenic hypothyroidism in the United States

Hashimoto thyroiditis

Hypothyroidism in the United States

Iatrogenesis

Power Review 275

Pyogenic osteomyelitis

Staphylococcus aureus

Neonatal septicemia and meningitis

Streptococcus agalactiae (Group B Streptococcus [GBS])

Sinusitis and otitis media in children

Pneumococcus

Meningitis in adults

Pneumococcus

Meningitis in renal transplantation patients

Listeria monocytogenes

Infantile bacterial diarrhea

Enteropathogenic Escherichia coli (Rotavirus is the MCC overall.)

Infantile diarrhea

Rotavirus

Chancre

Haemophilus ducreyi

Urethritis in a young, newly sexually active individual

Staphylococcus saprophyticus

Cystitis

Escherichia coli

Erysipelas

Streptococcus pyogenes

Chronic metal poisoning

Lead

Acute metal poisoning in the United States

Arsenic

Name the MCC of death: In neonates

Neonatal respiratoiy distress syndrome (NRDS)

In systemic lupus erythematosus (SLE)

Renal failure

In diabetic individuals

MI

In patients with cystic fibrosis

Pulmonaiy infections

276 USMLE Step I Recall

What is the MCC of death in the United States?

Heart disease

What is the second leading cause of death in the United States?

Cancer

What is the MCC of death in children aged 1-14 in the United States?

Accidents

What is the second leading cause of death in children aged 1-14 in the United States?

Cancer

What is the MCC of cancer mortality in males and females?

Lung cancer

What is the second leading cause of cancer deaths in males? In females?

For males it's prostate cancer, and for females it's breast cancer.

What is the MCC of death in black males aged 15-24?

Homicide. It is also the leading cause of death in black females aged 15-24.

What are the two MCCs of acute epididymitis in males: Younger than 35 years old?

Older than 35 years old?

Neisseria gonorrhoeae and Chlamydia trachomatis Escherichia coli and Pseudomonas sp.

What is the most common causative organism of acne vulgaris?

Propionibacterium acnes

What is the MCC of viral conjunctivitis?

Adenovirus

Power Review 277

What is the MCC of the following endocarditis scenarios: Postdental work?

Streptococcus viridans

Following biliary infections?

Ente rococcus faecal is

Non-IV drag user?

Staphylococcus aureus

Name the MCC: Epiglottitis in an unvaccinated child

Haemophilus influenzae type B

Pneumonia in adults

Streptococcus pneumoniae

Pneumonia in a neutropenic b u m patient

Pseudomonas sp.

Pneumonia in a patient with atypical bird exposure and hepatitis

Chlamydia psittaci

Walking pneumonia, seen in teens and military recruits

Mt/coplasma pneumoniae

Aspiration pneumonia in an alcoholic

Klebsiella pneumoniae

Pneumonia in an HIVpositive patient with CD4 0.8 Base : Most ventilated and most perfused : Blood flow exceeds O, delivery, resulting in underinflation. :V/Q ratio
View more...

Comments

Copyright ©2017 KUPDF Inc.
SUPPORT KUPDF